Download as pdf or txt
Download as pdf or txt
You are on page 1of 117

Item: 1 of 25 ~.

I • M k <:] t> al ~· ~
QIO: 1015 .l. ar Previous Next lab 'Vfl1ues Notes Calculator

•1 •
A 62 -year -old brea st cancer survivor post-radiation therapy visits her physician because of weakness, fatigue, fever; and weight gain. The physician
•2 elicits complaints of abdominal discomfort and progressive exertional dyspnea . Physical examination reveals hepatomegaly and jugular venous
distention that fail to subside on inspiration . Blood pressure is 115/70 mm Hg. An echocardiogram shows reduced biventricular end-diastolic volumes
•3 and elevat ed diastolic pressures in both ventricles.
•4
•5 Which of the following is the most likely diagnosis?

•6
A. Cardiac t amponade
•7
•8 B. Constrictive pericarditis

•9 c. Dilat ed cardiomyopathy
• 10
D. Recurrence of brea st cancer
· 11
E. Systolic congestive heart failure
• 12
• 13
• 14
• 15
• 16
• 17

• 18
• 19
• 20
• 21

Q
lock
s
Suspend
0
End Block
Item: 1 of 25 ~ 1 • M k -<:J 1>- Jil ~· !:';-~
QIO: 10 15 ..L ar Prev ious Next Lab~lues Notes Calculat o r

& &
1
•2
Th e correct an sw er i s B. 520/o chose this.
•3 The best answer is constrictive pericarditis because of the patient's post-radiation history, which is associated with thickening of the pericardium. Clinically,
cardiac tamponade and constrictive pericarditis have similar presentations, although cardiac tamponade typically presents with more acuity and
·4 hemodynamic instability. Constrictive pericarditis can also occur after trauma, neoplastic disease, and other chronic diseases.
•5 Cardiac tamponade Constnctive pericarditis Pericardium Pericarditis Hemodynamics Neoplasm

•6 A i s not correct. 1 3% chose t his•


Cardiac tamponade presents very similarly to constrictive pericarditis. In addition to Beck triad (distant heart sounds, jugular venous distention, and
•7 hypotension), pulsus paradoxus (a decrease in systolic pressure by 10 mm Hg during Inspiration) is usually present. These symptoms are typically the first
signs of tamponade.
·8
Cardiac tamponade Constrictive pericarditis Pulsus paradoxus Pericarditis Hypotens•on Heart sounds Jugular venous pressure cardiac cycle Systole B•ood P• ess~on e
.9 Jugula vein Vein
• 10 Cis not correct. 1 7 % ch ose this.
• 11 The echocardiogram results are not consistent with dilated cardiomyopathy, as diastolic volumes are reduced in this patient. In dilated cardiomyopathy,
end-diastolic volumes are increased .
• 12 Dilated cardiomyopathy Echocardiography Diastole Cardiomyopathy
• 13 0 is not co rrect. 2 0/o chose this.
• 14 Breast and lung carcinomas, lymphomas, and melanomas are the most common metastases to the pericardium and therefore should be considered In this
case . However, metastasis causing constrictive pathology is much less common than constrictive pericarditis .
• 15 Constrictive pericarditis Pericardium Pericarditis Metastasis Lung Lymphoma Pathology Melanoma Carcinoma
• 16 E i s not co rrect . 160/o ch ose this .
• 17 Systolic congestive heart failure (CHF) produces signs and symptoms similar to those In the vignette (weight gain, fatigue), but usually will not have a
fever. Also, systolic CHF would present with increa sed left ventricular end-diastolic volume .
• 18 Heart failure Congestive heart failure End- diastolic volume Systole Fatigue (medical)
• 19
• 20
Bottom Line:
• 21
• Constrictive oericarditis is marked bv oroliferation of fibrous tissue. with occasional small foci of calcification. that causes a loss of elasticity of oerlcardlal

a
Lock
s
Suspend
8
End Block
Item: 1 of 25 ~. I • M k <:] t> al ~· ~
QIO: 1015 .l. ar Previous Next lab 'Vfl1ues Notes Calculator
• Systolic congestive heart failure (CHF) produces signs and symptoms similar to those in the vignette (weight gain, fatigue), but usually will not have a •
1
fever. Also, systolic CHF would present with increased left ventricular end-diastolic volume .
•2 Heart failure Congestive heart failure End-diastolic volume Systole Fatigue (medical)

•3
•4 Bottom Line:
•5 Constrictive pericarditis is marked by proliferation of fibrous tissue, with occasional small foci of calcification, that causes a loss of elasticity of pericardiaI
tissues. It can mimic the signs of right heart failure, and can be caused by a variety of disease processes and infections such as systemic lupus
•6
erythematosus, rheumatic fever; post -radiation changes, and viral and bacterial infections.
•7 Constrictive pericarditis Systemic lupus erythematosus Rheumatic fever Pericarditis Pulmonary heart disease Congestive heart failure Heart failure Heart
Pericardium lupus erythematosus Fever Connective tissue Calcification Virus
•8
•9
• 10
lijl;fiiJI•l toryear:[2017 • ]
FI RST AID FAC T S
· 11
• 12
FA17 p 300.2
• 13
Acute pericarditis InAammation of the pericardium [fJ, red arrows]. Commonly presents with sharp pain, aggravated
• 14
by inspiration, and relieved by silting up and lean ing forward. Often complicated by pericardia!
• 15
effusion [between yellow arrows in fJ]. Presents with friction rub. ECG changes include
• 16 widespread ST-segment elevation and/or PR depression.
• 17 Causes include idiopathic (most common; presumed viral), confirmed infection (eg,
• 18 Coxsackievirus), neoplasia, autoimmune (eg, SLE, rheumatoid arthritis), uremia, cardiovascular
• 19
(acute STEM! or Dressler syndrome), radiation therapy.

• 20
• 21

6
lock
s
Suspend
0
End Block
IC

Coxsackievirus), neoplasia, autoimmune (cg, SLE, rheumatoid arthritis), uremia, cardiovascu lar
•2
(acute STE~1I1 or Dressler syndrome), radiation therapy.
•3

·4
•5
•6 FA17 p 300.3

•7 Cardiac tamponade Compression of the heart b} Auid (eg. blood, effusions (arrows in r.JJin pericardial space) - l CO.
·8 Equilibration of diastolic pressures in all 4 chambers.
Findings: Beck triad (hypotension, distended neck ,·eins, distant heart sounds), t H R, pulsus
~

.
.9
paradoxus. ECC shows low-,·oltage QRS and electrica l alternans (due to "swinging" movement of
~~ I
·';.:·:~'
• 10
heart in large effusion).

--~~ ~
• 11
Pulsus paradoxus- 1 in amplitude of systolic BP by> 10 mm Hg during inspiration. Seen in
• 12
cardiac tamponade, asthma, obstructi\C sleep apnea, pericarditis, croup.
• 13
• 14
- ..
• 15
FA17 p 298.1
• 16
Heart failure C linical syndrome of cardiac pump dysfun ction - congestion and low perfusion. Symptoms
• 17
include dyspnea, orthopnea, fatigue; signs include $3 heart sound, rales, jugular venous distention
• 18
(JVD), pitting edema · .
• 19 Systolic dysfunction-reduced EF, t EDV; l contractility often 2° to ischemia/MI or dilated
• 20 cardiomyopathy.
• 21 Diastolic dysfunction-presen·ed E I<~ normal EDV; l compliance often 2° to myocardial

a
Lock Suspend
s 8
End Block
Item: 2 of 25 ~. I • M k <:] t> al ~· ~
QIO: 1034 .l. ar Previous Next lab 'Vfl1ues Notes Calculator

1 •
A 32 -year -old man with diabet es mellitus type 1 presents to his physician stating that he sometimes "passes out." He denies a t emporal pattern and
•2 stat es that these episodes are associat ed with dizziness. The patient has a blood pressure of 136/76 when sea t ed and 114/70 while standing. An
alteration is suspected in the normal physiologic response carried out by arterial baroreceptors locat ed in the aortic arch and the carotid sinus.
•3
•4 What is the normal physiologic response of the baroreceptors to hypotension?
•5
•6 A. Decrea sed baroreceptor afferent firing in the aortic arch lea ds to increa sed parasympathetic efferent firing
•7
B. Decrea sed baroreceptor afferent firing in the carotid sinus lea ds to increa sed sympathetic efferent firing
•8
c. Increa sed baroreceptor afferent firing in the aortic arch lea ds to decrea sed parasympathetic efferent firing
•9
• 10 D. Increa sed baroreceptor afferent firing in the aortic arch lea ds to increa sed sympathetic efferent firing

· 11 F. ln~rP.i'I SP.rl h;~ rorP. ~P. ptor i'lffP.rP.nt firing in thP. ~;~ rotirl sinus IP.;~ rls to in~rP.i'I SP.rl symp;~ th P.ti~ P.ffP.rP.nt firing
• 12
• 13
• 14
• 15
• 16
• 17

• 18
• 19
• 20
• 21

lock
Q s
Suspend
0
End Block
2
•3
The correct a n sw er i s B. 690/o chose this.
·4
The carotid sinus baroreceptor sends an afferent signal via the glossopharyngeal nerve to the medulla, which in turn responds by increasing sympathetic
•5 outflow. This results 1n systemic vasoconstriction, increased heart rate, Increased contractility, and increased blood pressure. In contrast to the aortic arch
baroreceptors, which primarily respond to higher blood pressures of 110-180 mm Hg, the carotid baroreceptors are capable of transdudng mechanical
•6 stresses at pressures from 50- 180 mm Hg and are thus more responsive to hypotension. In a hypotensive state, the baroreceptor afferent firing rate
•7 decreases, which leads to increased sympathetic efferent firing and subsequent increase in blood pressure.
Baroreceptor Carotid sinus Glossopharyngeal nerve Hypotensoon vasoconstriction Ao tiC arch Blood pressure Heart rate Afferent nerve fiber Medulla oblongata
·8 Cont acti ity Efferent nerve fiber Tachycardia carotid artery Common carotid artery Sympathetic nervous system
.9 A i s not correct. 80/o chose this.
• 10 Although the baroreceptor afferent firing does decrease, the parasympathetic efferent firing is subsequently decreased, not increased. If parasympathetic
efferent firi ng were to increase, it would stimulate the cholinergic parasympathetic innervation of the heart, causing a decrease in the discharge rate of
• 11 the sinoatrial node and slowing of the ventricular conduction system, which In turn would further worsen hypotension.
• 12 Sinoatrial node Baroreceptor Cholinergic Hypotension Parasympathetic nervous system Afferent nerve fiber Efferent nerve fiber

• 13 C is not co rrect. 80/o chose t his •


Although parasympathetic efferent firing is decreased in a hypotensive state, afferent baroreceptors in the aortic arch would exhibit decreased firing, not
• 14 Increased firing . Hypotension results in decrea sed baroreceptor afferent firing, which leads to increased sympathetic firing ; this increases contractility,
systemic vasoconstriction, and heart ra t e to increa se blood pressure. Although hypotension would lead to decrea sed parasympathetic efferent firing, the
• 15
activation of the sympathetic system is primarily what accounts for the compensatory increase in blood pressure.
• 16 Baroreceptor Vasoconstriction Aortic arch Hypotension Parasympathetic nervous system Sympathetic nervous system Blood pressure Heart rate
Efferent nerve fiber Afferent nerve fiber Contractility
• 17
• 18 D is not co rrect. 80/o ch ose this .
The baroreceptor afferent firing rate is stretch-dependent and therefore pressure sensitive; it would decrease, not increase, with hypotension .
• 19 Baroreceptor Hypotension Afferent nerve fiber
• 20 E i s not correct. 5% chose this.
• 21 The baroreceptor afferent firing rate would decrease with hypotension. On the contrary, an increase in blood pressure increases afferent firing rate .

a
Lock
s
Suspend
8
End Block
Item: 2 of 25 ~. I • M k <:] t> al ~· ~
QIO: 1034 .l. ar Previous Next lab 'Vfl1ues Notes Calculator
.._. .......................,
1
D is no t co rrect. 8 % cho se this.
2 The baroreceptor afferent firing rate is stretch-dependent and therefore pressure sensitive; it would decrease, not increase, with hypotension.
•3 Baroreceptor Hypotension Afferent nerve fiber

.4 E is no t co rrect. 5 % cho se this.


The baroreceptor afferent firing rate would decrease with hypotension. On the contrary, an increase in blood pressure increases afferent firing rate.
•5
Baroreceptor Hypotension Blood pressure Afferent nerve fiber
•6
.7
Bo tto m Line:
•8
The carotid sinus contains baroreceptors that synapse through cranial nerve IX in the nucleus solitarius of the medulla to mediate the autonomic
•9 nervous system. The carotid body is the location of numerous chemoreceptors .
Solitary nucleus Autonomic nervous system Carotid sinus Carotid body Cranial nerves Synapse Glossopharyngeal nerve Medulla oblongata Nervous system
• 10 Baroreceptor Chemoreceptor Common carotid artery Carotid artery
· 11
• 12
• 13 lijj ;fi IJ l•l f o r year:l 2 0 1 7 ..
FI RST AI D FA CTS
• 14
• 15
FA17 p 285.3
• 16
Baroreceptors and chemoreceptors Receptors:
• 17
AFFERENT EFFERENT
• Aortic arch transmits via vagus nerve to sol itary nucleus of
• 18 medulla (responds to ! and t in BP).
Solitary nucleus ~
• 19 • Ctuotid sinus (dilated region at carotid hi furcation) transmits via
• 20
• 21
n=~~L O
/" Sympathetic
chain
glossopharyngeal nerve to solitary nucleus of medulla (responds
to ! and t in BP).
IV• .... ______ ---· -··-·
6
lock
s
Suspend
0
End Block
2 Chemoreceptors:
Peripheral-carotid and aortic bodies are stimulated by l Po2
•3
(< 60 mm Hg), t Pco2, and l pH of blood.
·4 Central-arc ~timulated by changes in pH and Pco1 of brain
•5 interstitial Auid. which in turn are influenced by arterial C02.
•6 Do not directlr respond to Po2.
•7

·8 FA17 p 286.2
.9 Autoregulation How blood Row to an organ remains constant over a wide range of perfusion pressures.
• 10
ORGAN FACTORS DETERMINING AUTOREGULATION
• 11
Heart Local metabolites (vasodih1tory): adenosine, Note: the pulmonary vasculature is unique in
• 12 0, C02, l 0 2 that hn>oxia causes vasoconstriction so that
• 13 Brain Loca l metabol ites (vasodilalorr): C02 (pi I) only well-ventilated areas arc perfused. In
• 14 other organs, hypoxia causes vasod ilation .
Kidneys Myogenic and tubuloglomerular feedback
• 15
Lungs Hypoxia causes vasoconstriction
• 16
Skelet al muscle Local metabol ites during exercise: lactate, C0 2, JJ+, Adenosine, Lactate, K+ (C IIALK)
• 17 adenosine, K+, 1-f+, C0 2
• 18 At rest: srmpathetic tone
• 19 Skin Srmpathetic stimulation most important
• 20 mechanism for temperature control
• 21

a
Lock
s
Suspend
8
End Block
Item: 3 of 25 ~ 1 • M k -<:J 1>- Jil ~· !:';-~
QIO: 10 14 ..L ar Pre v ious Next Lab~lues Notes Calcula t o r

1
&

A 54-year-old man comes to the emergency department claiming that he Is having a heart attack. He complains of intermittent chest pain that has
continued over the past 6 hours. The pain is worse when he lies down. He denies having any previous types of these episodes. He has no tobacco use
IA•A] &

2
history and drinks one glass of wine with dinner daily. On review of systems, he denies any shortness of breath or diaphoresis. His bloodwork Is drawn,
•3 and his laboratory results are as follows:
·4 WBC: 6.7 K/ mm3
RBC: 4.7 M/ mm3
•5 Hemoglobin: 15.1 g/ dl
•6 Hematocrit: 44.6o/o
Platleys: 250 K/mm3
•7 Troponln: <0.01 ng/ ml
CK-MB: 4.3 ng/ ml (normal range: s 7.7 ng/ ml, males)
·8
An ECG Is also obtained and is shown.
.9
• 10
• 11
• 12
• 13
• 14
• 15
• 16
• 17
• 18
• 19
• 20
• 21

a
Lock
s
Suspend
8
End Bl ock
Item: 3 of 25 ~ 1 • M k -<:J 1>- Jil ~· !:';-~
QIO: 10 14 ..L ar Pre v ious Next Lab~lues Notes Calcula t o r
~

1
; 4
9 9 9
An ECG Is also obtained and is shown.
2
l
•3
.A.t--+--H,r.:.-t-~'.r-+-'--+1 t:::::=._:...,~,---.-
·4 1-
•5
•6
.7
·8
.9
• 10
• 11

• 12
• 13
• 14
• 15
• 16
• 17 Image courtesy of CardioNetworks
. 18
• 19 Which of the following is the most likely cause of this man's symptoms?

• 20
A. Asthma
• 21

a
Lock
s
Suspend
8
End Bl ock
Item: 3 of 25 ~ 1 • M k -<:J 1>- Jil ~· !:';-~
QIO: 10 14 ..L ar Pre v ious Next Lab~lues Notes Calcula t o r
& &
1
2
•3

·4
•5
•6
•7

·8
.9
• 10
• 11
• 12
Which of the following is the most likely cause of this man's symptoms?
• 13
• 14 A. Asthma
• 15
B. Gastroesophageal disease
• 16
• 17 c. Myocardial infarction

• 18 D. Pericarditis
• 19
E. Pulmonary embolism
• 20
• 21

a
Lock
s
Suspend
8
End Bl ock
Item: 3 of 25 ~ 1 • M k -<:J 1>- Jil ~· !:';-~
QIO: 10 14 ..L ar Previous Next Lab~lues Notes Calculat or
& &
1 Th e co rrect a n sw er i s B. 5 9 0/o chose this.
2 The patient presents to the emergency department complaining of chest pain, and a cardiac cause would be the most concerning . However, given the
absence of respiratory symptoms, negative cardiac enzymes, or any of the following ECG changes: ST elevation/depression, T wave inversion, or
3 development of Q waves, the m ost likely diagnosis for this patient is a noncardlac disease such as gastroesophageal reflux disease (GERD). GERD Is
·4 caused by decreased lower esophagea l sphincter tone and classica lly manifests with chest pain following meals and is worse when laying down.
Gastroesophageal reflux disease Chest pain Electrocardiography QRS complex T wave Lower esophageal sphincter cardiac marker Emergency department Card1a
•5 Sphincte•
•6
A i s not correct. 1 % chose t his.
.7 Patients with acute asthma attacks often present with shortness of breath, wheezing, tachycardia, and cough, which may be mistaken for pulmonary
embolus or anxiety attack if a history of asthma is unclear. It is unlikely that an acute asthma attack is suspected of being a myocardial infarction.
·8 Pulmonary embolism Myocardial infarction Asthma Tachycardia Pamc attac Embolu Dyspnea Embolism Cough Anxiety Wheeze Infarction
.9 C i s not c orrect. 7 % chose this.
• 10 The patient's cardiac enzyme leve ls do not indicate acute myocardial Infarction, and the ECG shows normal sinus rhythm. ECG changes for a myocardial
Infarction can Involve ST elevation if a transmural infarction is present or ST depression if a subendocardial infarction is present. Other signs of a
• 11 myocardial infarction on ECG include hyperacute T waves, pathologic Q waves, poor R-wave progression, a new left bundle branch block, or T-wave
• 12 Inversions (the normal pattern is for the T waves to be upright in all leads except for aVR and v 1 ) .
Left bundle branch block Myocardial infarction Electrocardiography Bundle branch block QRS complex Sinus rhythm Enzyme ST elevation ST depression T wave
• 13 Love wave Major depressive disorder Coronary circulation
• 14
D is not co rrect . JOO/o chose this.
• 15 The symptoms of gastroesophageal reflux disea se (GERD) have substantial overlap with pericarditis. Pericarditis m ay also m anifest with pain that radiates
to the back, trapezius ridge, shoulders, and neck. Although this patient's pain Isn't definitively distinguished from pericarditis, pericarditis would more
• 16 classically manifest with chest pain that improves with leaning forward, a friction rub on physical exam, and diffuse ST elevation and reciprocal PR
• 17 depression In a majority of leads on ECG.
Gastroesophageal reflux disease Pericarditis ST elevation Trapezius muscle Chest pain Physical examination Pericardia! friction rub Depression (mood)
. 18
E i s n ot co rrect. 10/o ch ose this .
• 19 Pulmonary embolism is sometimes mistaken for a heart attack, but not as frequently as gastroesophageal diseases. Although a patient with a pulmonary
• 20 embolus (PE) would also present with chest pain, and likely a normal ECG, the chest pain in a PE is typically inspiratory (or pleuritic) and does not change
with position. Additiona lly, a PE typically occurs in the setting of specific risk factors: immobilization, recent surgery, recent extended airplane flight, etc.
• 21 This patient does not have any of those risk factors, making a PE very unlikely.

a
Lock
s
Suspend
8
End Block
Item: 3 of 25 ~. I • M k <:] t> al ~· ~
QIO: 1014 .l. ar Previous Next lab 'Vfl1ues Notes Calculator
.. w ... ... ... .. ... -

1 Gastroesophageal reflux disease Pericarditis ST elevation Trapezius muscle Chest pain Physical examination Pericardia! friction rub Depression (mood)

2 E is not correct. 1 Ofo chose this .

3 Pulmonary embolism is sometimes mistaken for a heart attack, but not as frequently as gastroesophageal diseases. Although a patient with a pulmonary
embolus (PE) would also present with chest pain, and likely a normal ECG, the chest pain in a PE is typically inspiratory (or pleuritic) and does not change
.4 with position. Additionally, a PE typically occurs in the setting of specific risk factors : immobilization, recent surgery, recent extended airplane flight, etc.
This patient does not have any of those risk factors, making a PE very unlikely.
•5
Pulmonary embolism Embolism Electrocardiography Embolus Pleurisy Myocardial infarction Chest pain
•6
.7
Bottom Line:
•8
Gastroesophagea l diseases can result in epigastric pain that mimics the symptoms of a myocardial infarction .
•9 Myocardial infarction Abdominal pain Epigastrium Infarction

• 10

· 11
• 12 lijj ;fi IJ l•l for year:l 2017 ..
FIRST AID FAC T S
• 13
• 14 FA17 p295.1
• 15 Diagnosis of In the first 6 hours, ECC is the gold standard.
• 16 myocardial infarction Cardiac troponin I rises after 4 hours (peaks
• 17 at 24 hr) and is t for 7-10 days; more specific
• 18
than other protein markers.
CK-MB rises after 6-12 hours (peaks at
• 19
16-24 hr) and is predominantly found
• 20 in myoca rdium but can also be released
• 21 from skeletal muscle. Useful in diagnosing Normal
'
6
lock
s
Suspend
0
End Block
Item: 3 of 25 ~ 1 • M k -<:J 1>- Jil ~· !:';-~
QIO: 10 14 ..L ar Pre v ious Next Lab~lues Notes Calcula t o r

1
&
pathologic Q waves or poor R W <l\ e progression &

2 (emlving or old transmural infarct).


3
·4 FA17 p 360.2
•5 Esophageal pathologies
•6 Boerhaave syndrome Transmural, usually distal esophageal rupture'' ith pneumomediastinum {arrows in ) due to
.7 \'iolent retching. Subcutaneous emph}sema ma) be due to dissecting air (crepitus may be felt in
the neck region or chest wall). urgical emergency.
·8
.9
Eosinophilic Infiltration of eosinophils in the esophagus often in atopic patients. Food allergens - dysphagia,
esophagitis food impaction. Esophageal rings and linear furrows often seen on endoscopy. Unresponsive to
• 10
CERD therapy.
• 11
Esophageal strictures Associated with caustic ingestion and acid reflux.
• 12
Esophageal varices Dilated submucosal veins (red arrows) : B in lo\\ er 11> of esophagus (white arrow) 2° to portal
• 13
hypertension. Common in cirrhotics, may be source of life-th reatening hematemesis.
• 14
Esophagitis Associated with reflux, infection in immunocon1pr01niscd (Candida: white pseudomembrane;
• 15
IISV-1: punched-out ulcers; CMV: linear ulcers), caustic ingestion, or pill esophagitis
• 16 (eg, bisphosphonates, tetracycl ine, SAl Ds, iron, and potassium chloride) .
• 17 Gastroesophageal Commonly presents as heartburn, regurgitation, dy~phagia. lay also present as chronic cough,
. 18 reflux disease hoarseness {laryngopharyngea l reAux). Associated with asthma. Transient decreases in LES tone.
• 19 Mallory-Weiss Partial-thickness mucosal lacerations at gastroesophageal junction due to se\'ere ,·omiting. Often
• 20 syndrome presents with hcmatemesis and may be misdiagnosed as ruptured esophageal varices. Usually
• 21
found in alcoholics and bulimics.

a
Lock
s
Suspend
8
End Bl ock
Item: 4 of 25 ~. I • M k <:] t> al ~· ~
QIO: 1035 .l. ar Previous Next lab 'Vfl1ues Notes Calculator

1 •
An 84-year -old Ca ucasian man has a history of coronary artery disea se. He presents with a primary complaint of chest pain that is associat ed with
2 activity and is progressively worsening. He has a 35 pack-year history of cigarette smoking and had an abdominal aortic aneurysm repaired several
years ago. His father died of a stroke at age 80. He is prescribed metoprolol (a j3 -blocker) to trea t his condition.
3
•4 Mean Arterial
Choice Heart Rate (bpm) Stroke Volume (ml) Pressure (mmHg)
•5 A -J, 1' -J,
•6 B 1' 1' 1'
•7
c -J, 1' 1'
0 1' -J, No Change
•8 1'
E No Change 1'
•9
• 10
Using the t able above, what effects will metoprolol have on heart ra t e ( HR), stroke volume (SV), and mean arterial pressure (MAP)?
· 11
• 12 A
• 13
B
• 14
• 15
c
• 16 D

• 17 E
• 18
• 19
• 20
• 21

6
lock
s
Suspend
0
End Block
Item: 4 of 25 ~ 1 • M k -<:J 1>- Jil ~· !:';-~
QIO: 10 35 ..L ar Prev ious Next Lab~lues Notes Calculat o r

& &
1 Th e co rrect an sw er i s A. 7 40/o ch ose this.
2 Metoprolol has a negative chronotropic effect on the heart and thus decreases the heart rate . As a class, 13-blockers also have a negative inotropic effect,
decreasing contractility, but do not confuse this with stroke volume (SV). CO = HR x sv. 13-Biockers decrease heart rate (HR), but even so, cardiac output
3 (CO) only decreases by 5%-14%. The mild decrease in co is due to a dramatic increase in stroke volume (SV). sv = End-diastolic volume (EDV)- end-
systolic volume (ESV). The decrease in HR allows for increased diastolic filling, thus increasing EDV and preload. sv is intrinsically controlled by preload
4
(the degree to which the ventricles are stretched prior to contracting). The mean arterial pressure (MAP) is the average over a cardiac cycle and Is
•5 determined by the CO, systemic vascular resistance (SVR), and central venous pressure (CVP). In practice the contribution of CVP (which is small) Is
generally Ignored. MAP= cox SVR. Since co is decreased by 5%-14%, the MAP will slightly decrease.
•6 Metop olo I ootrope Cardiac cycle Chronotropic Stroke volume Mean arterial p ess e Central •enous pressure cardiac output Vascular resistance

•7 End-ststolic volume Heart rate Diastole Contractility Ventricle (heart) Preload (cardiology) Vein Blood vessel Stroke

·8 B i s not correct. 4 % chose this.

.9 This scenario would be seen if the patient was given medication such as epinephrine. Stimulation of 131 -receptors by epinephrine induces a positive
chronotropic and inotropic effect on the heart. Antagon ism of the receptors by 13-blockers results in reduction in heart rate, stroke volume, and mean
• 10 arterial pressure .
Epinephrine Chronotropic lnotrope Stroke volume Mean arterial pressure Bradycardia Heart rate Pharmaceutical drug Stroke Receptor (biochemistry)
• 11
Blood pressure
• 12
C is not co rrect. 150/o chose this •
• 13 This combination of changes in heart rate, stroke volume, and mean arterial pressure are not consistent with administration of a 13-blocker. These changes
would most likely be seen with digoxin .
• 14
Digoxin Stroke volume Mean arterial pressure Heart rate Stroke Blood pressure
• 15
D is no t co rrect . 60/o ch ose this .
• 16 This combination of findings (elevated heart ra t e and decreased stroke volume) without a change in mean arterial pressure are not consistent with
administration of a 13-blocker.
• 17
Stroke volume Mean arterial pressure Heart rate Stroke
• 18
E i s n ot co rrect. 10/o ch ose this .
• 19 This combination of changes (no change in heart rate, increased stroke volume, and Increased mean arterial pressure) are not consistent with
administration of a 13-blocker.
• 20
Stroke volume Mean arter1al pressure Heart rate Stroke
• 21

a
Lock
s
Suspend
8
End Block
Item: 4 of 25 ~. I • M k <:] t> al ~· ~
QIO: 1035 .l. ar Previous Next lab 'Vfl1ues Notes Calculator
... ... ... - - ...
1 Stroke volume Mean arterial pressure Heart rate Stroke
2 E is not correct. 1 Ofo chose this .
3 This combination of changes (no change in heart rate, increased stroke volume, and increased mean arterial pressure) are not consistent with
administration of a 13-blocker.
4 Stroke volume Mean arterial pressure Heart rate Stroke
•5
•6
Bottom Line:
.7 13-Biockers have negative chronotropic and inotropic effects on the heart, leading to decreased heart rate and mean arterial pressure. However;
•8 decreased heart rate allows for increased diastolic filling, resulting in increased EDV and preload, ultimately increasing stroke volume .
Chronotropic Inotrope Mean arterial pressure Stroke volume Heart rate Diastole Preload (cardiology) Stroke
•9
• 10

· 11 I ill ;fi 1!1 I•J for year:l 2017 ..


FI RST AID FA CTS
• 12
• 13
FA17p237.1
• 14
p-blockers Acebutolol, atenolol, betaxolol, bisoprolol, carvedilol, esmolol, labetalol, metoprolol, nadolol,
• 15
nebivolol, pindolol, propranolol, timolol.
• 16
APPLICATIOtl ACTIONS tiOTES/EXAMPLES
• 17
Angina pectoris ! heart rate and contractility, resulting in ! 0 2
• 18 consumption
• 19 Myocardial infarction ! mortality
• 20 Supraventricular ! AV conduction ,·elocity (class II Metoprolol, esmolol
• 21 tachycardia antiarrhythmic)

6
lock
s
Suspend
0
End Block
Item: 4 of 25 ~ 1 • M k -<:J 1>- Jil ~· !:';-~
QIO: 10 35 ..L ar Pre v ious Next Lab~lues Notes Calcula t o r
& &
1
FA17 p 309.1
2
Antiarrhythmics- Metoprolol, propranolol, esmolol, alenolol, timolol, carvedilol.
3
P-blockers (class II)
4
MECHANISM Decrease SA and A nodal acti' ily b) l cA~ I P, l Ca 2+ currents. Suppress abnormal pacemakers b)
•5
l slope of phase 4.
•6 AV node particularly sensiti,·e- t PR interval. Esmolol ,·err short acting.
.7 CLINICAL USE SVT, ventricular rate control for atrial fibrillation and atrial Autter.
·8 ADVERSE EFFECTS Impotence, exacerbation of COPD and asthma, cardiovascular effects (bradycardia, AV block, IfF),
.9 C 1 S effects (sedation, sleep alterations). .\!lay mask the signs of hypoglycemia.
• 10 l\ letoprolol can cause dyslipidemia. Propranolol can exacerbate ' 'asospasm in Prinzmetal angina .
• 11 ~ -blockers (except the nonselective a- and ~-antagoni sts can·cdilol ancllabetalol) cause unopposed
<Xragonism if gi,·en alone for pheochromocytoma or cocaine toxicity. Treat ~-blocker overdose" ith
• 12
saline, atropine, glucagon .
• 13
Class II
• 14 Oeoease slope Prolonged
orphase 4 repolarization
• 15 depolarization (at /node)
• 16
• 17
. 18
• 19 ,....---
100 200 300 400 soo 600 700
• 20 Time lmsl
Pacemaker cell actiOn potential 13
• 21

a
Lock
s
Suspend
8
End Bl ock
Item: 4 of 25 ~ 1 • M k -<:J 1>- Jil ~· !:';-~
QIO: 10 35 ..L ar Pre v ious Next Lab~lues Notes Calcula t o r

1
~ -60J-_ _ . : : - - l . - - - -
2 ~
-90 ,----
3 0 100 200 300 400 500 600 700
nme tmsl
4 Pacemaker cell act1011 potential
•5
•6 FA17 p 272.1
•7
Cardiac output CO= stroke volume (SV) x heart rate (IIR) During the early stages of exercise, CO is
·8
Fick principle: maintained b) t HR and t SV. During the late
.9 stages of exercise, CO is maintained by t HR
CO = rate of0 7 consumption
• 10 only (SV plateaus).
arterial 0 2 content - ,·cnous 0 2 content
Diastole is preferentially shortened with t II R;
• 11
Mean arterial pressure (M P) =CO x tol<li less fi lling time .... l CO (cg, ventricular
• 12
peripheral resistance (TPR) tachycardia).
• 13
MAP= 2/, diastolic pressure+ Xsystolic pressure
• 14
• 15 Pulse pressure= systolic pressure - diastolic pressure t pulse pressure in hyperthyroidism, aort ic
Pulse pressure is proportional to SV, inversely regurgitation, aortic stiffeni ng (isolated systolic
• 16
proportional to arterial compliance. hypertension in elderly), obstructive sleep
• 17
apnea (t sympathetic tone), exercise (transient).
• 18
S = end-<liastolic volume (ED ) - end-srstolic
l pulse pressure in aortic stenosis, cardiogcnic
,·olume (ESV)
• 19 shock, cardiac tamponade, advanced heart
• 20 fai lure (H F).
• 21

a
Lock Suspend
s 8
End Bl ock
Item: 5 of 25 ~ 1 • M k -<:J 1>- Jil ~· !:';-~
QIO: 10 39 ..L ar Pre v ious Next Lab~lues Notes Calcula t o r
& &
1
A 62-year-old male is admitted to the intensive care unit with a blood pressure Is 80/30 mm Hg and heart rate of 120/min. on examination, the
2 patient Is unresponsive, and his skin is cold and clammy.

3
A pharmacologic agent acting by which mechanism would allow for immediate compensation in this patient?
4
•5
A. Activation of the renin-angiotensin-aldosterone system
•6
•7 B. Parasympathetic stimulation of heart rate

·8 c. Release of vasopressin
.9
D. Stimulation of lh-receptors on vascular smooth muscle
• 10
E. Stimulation of sympathetic o 1 -receptors on vascular smooth muscle
• 11
• 12
• 13
• 14
• 15
• 16
• 17
• 18
• 19
• 20
• 21

a
Lock Suspend
s 8
End Bl ock
Item: 5 of 25 ~. I • M k <:] t> al ~· ~
QIO: 1039 .l. ar Previous Next lab 'Vfl1ues Notes Calculator

1 The co rrect a nswer is E. 68% cho se this.


2 Sympathetic o 1 -stimulation of the veins causes venoconstriction, resulting in decrea sed venous compliance (another t erm for venous pooling), thus
increa sing venous return to the heart . This would in turn increa se the preload and the resulting arterial pressure. Activation of the o 1 -receptors on
3 arteries and arterioles lea ds to vasoconstriction and increa sed total peripheral resistance, which in turn increa ses blood pressure. Trea tment with pressors
such as norepinephrine will raise blood pressure.
4 Vascular resistance Norepinephrine Vasoconstriction Antihypotensive agent Blood pressure Arteriole Preload (cardiology) Venous return curve Artery
5 Compliance (physiology) Vein

•6 A is no t co rrect. 10 % cho se this.


.7 The renin-angiotensin-aldosterone system is an effective way of increa sing arterial pressure by increa sing volume through the renal ret ention of fluids.
However; this is not a quick response mechanism because of the delay for t ra nsmission of a hormonal signal.
•8 Renin-angiotensin system Hormone Blood pressure

•9 B is no t co rrect. 4 % cho se this.


• 10 Parasympathetic stimulation of the heart would result in a decrea sed heart rat e and a corresponding decrea se in arterial pressure .
Parasympathetic nervous system Heart rate Blood pressure
· 11
c is no t co rrect. 9 % cho se this •
• 12 Vasopressin, or antidiuretic hormone (ADH ), is involved in the regulation of blood pressure in response to hemorrhage via two mechanisms. The first is via
• 13 the V 1 receptors found on vascular smooth muscle of the systemic, splanchnic, and coronary circulations (pressor activity). The second is via the V2
receptors locat ed mainly at the distal tubule and collecting ducts of the kidney (antidiuretic activity). However; these systems are not as quick as
• 14 sympathetic stimulation of o 1 -receptors. The onset of vasopressin's pressor activity is 30- 60 minutes, and the onset of its antidiuretic activity is 2 - 8 hours
when given intravenously. Because its onset is so slow, ADH is not a first-line pressor agent .
• 15 Distal convoluted tubule Vasopressin Kidney Smooth muscle tissue Vasoconstriction Blood pressure Intravenous therapy Antihypotensive agent Hormone
• 16 Vascular smooth muscle Collecting duct system Bleeding Receptor (biochemistry) Antidiuretic Anatomical terms of location Nephron Muscle

• 17 Sympathetic nervous system Blood vessel

• 18 D is no t co rrect. 9 % cho se this •


Sympathetic activation of !32 -receptors on va scular smooth muscle would cause vasodilation of the arterioles serving the skelet al muscles. This does not
• 19
lea d to an increa se in systemic blood pressure.
• 20 Vasodilation Sympathetic nervous system Smooth muscle tissue Blood pressure Arteriole Vascular smooth muscle Skeletal muscle Muscle Blood vessel

• 21

6
lock
s
Suspend
0
End Block
Item: 5 of 25 ~. I • M k <:] t> al ~· ~
QIO: 1039 .l. ar Previous Next lab 'Vfl1ues Notes Calculator

1 sympathetic stimulation of a 1 -receptors. The onset of vasopressin's pressor activity is 30- 60 minutes, and the onset of its antidiuretic activity is 2 - 8 hours
when given intravenously. Because its onset is so slow, ADH is not a first-line pressor agent.
2 Distal convoluted tubule Vasopressin Kidney Smooth muscle tissue Vasoconstriction Blood pressure Intravenous therapy Antihypotensive agent Hormone
3 Vascular smooth muscle Collecting duct system Bleeding Receptor (biochemistry) Antidiuretic Anatomical terms of location Nephron Muscle

4 Sympathetic nervous system Blood vessel

5 D is not correct. 9 % chose this .


Sympathetic activation of !32 -receptors on vascular smooth muscle would cause vasodilation of the arterioles serving the skeletal muscles. This does not
•6
lead to an increase in systemic blood pressure.
.7 Vasodilation Sympathetic nervous system Smooth muscle tissue Blood pressure Arteriole Vascular smooth muscle Skeletal muscle Muscle Blood vessel

•8
•9 Bottom Line:
• 10 Activation of a 1 -receptors causes vascular smooth muscle contraction, increasing blood pressure the quickest .
Smooth muscle tissue Blood pressure Muscle contraction Vascular smooth muscle Blood vessel
· 11
• 12
• 13
lijj ;fi IJ l•l for year:l 2017 ..
• 14 FIRST AID FAC T S

• 15
FA17 p273.1
• 16
Cardiac output variables
• 17
Stroke volume Stroke Volume affected by Contractility, SVCAP.
• 18
Afterload, and Preload. A failing heart has ~ SV (systolic and/or diastolic
• 19 t SV with: dysfunction)
• 20 • t Contractility (eg, anxiety, exercise)
• 21 • t Preload (eg, early pregnancy)

6
lock
s
Suspend
0
End Block
Item: 5 of 25 ~. I • M k <:] t> al ~· ~
QIO: 1039

1
.l. ar Previous
~
Next
~

contractility.
lab 'Vfl1ues
. , ~
Notes
' ~
Calculator
. .
2
3
FA17p234.1
4 Sympathomimetics
5 DRUG ACTION APPLICATIONS
•6
Direct sympathomimetics
•7
Albuterol, salmeterol ~z > ~ 1 Albuterol for acute asthma or COPO. Salmeterol
•8 for long-term asthma or COPO control.
•9 Dobutamine ~1 > 13z, a Heart fail ure (llF) (inotropic > chronotropic),
• 10 cardiac stress testing.
· 11 Dopamine 0 1= 0 2> ~> a Unstable bradycardia, HF, shock; inotropic and
• 12 chronotropic effects at lower doses due to ~
• 13 effects; vasoconstriction at high doses due to a
effects.
• 14
Epinephrine ~> a Anaphylaxis, asthma, open-angle glaucoma;
• 15
a effects predominate at high doses .
• 16
Significantly stronger effect at 13z-receptor than
• 17 norepinephrine .
• 18 Fenoldopam 01 Postoperative hypertension, hypertensive crisis.
• 19 Vasodilator (coronary, periphera l, renal, and
• 20 splanchnic). Promotes natriuresis. Can cause
• 21 hypotension and tachycardia .

6
lock
s
Suspend
0
End Block
Item: 6 of 25 ~ 1 • M k -<:J 1>- Jil ~· !:';-~
QIO: 10 11 ..L ar Pre v ious Next Lab~lues Notes Calcula t o r
& &
1
A 24-year-old female presents to her primary care physician because she feels as if her heart is intermittently racing . She states that she recently felt
2 dizzy and passed out. She has experienced similar episodes in the past. On examination, her blood pressure is 105/70 mm Hg and her heart rate is
115/min. Cardiovascular exam demonstrates t achycardia, with a regular rhythm, normal 51 and 52, and no 53 or 54. Pulmonary exam demonstrates
3 crackles in the lungs bilaterally. The patient's extremities are cool. A complete blood count, chemistry panel, liver function tests, and thyroid panel are ali
within normal limits. An ECG is performed and the lead II tracing is shown.
4
5
~:
•6 [+ . ~· ....
•7 :Ht
H;+t+t ·+
..;.
·8
. I

.9
• 10
• 11
What is the cause of this patient's condition?
• 12
• 13 A . Atrial fibrillation
• 14
B. Atrioventricular bypass tract
• 15
• 16
c. Atrioventricular nodal defect

• 17 D. Bundle-branch block

• 18 E. His-Purkinje defect
• 19
• 20
• 21

a
Lock
s
Suspend
8
End Bl ock
Item: 6 of 25 ~ 1 • M k -<:J 1>- Jil ~· !:';-~
QIO: 1011 ..L ar Prev ious Next Lab~lues Notes Calculat o r

& &
1 Th e co rrect an sw er i s B. 510/o chose this.
2 This patient's ECG is characteristic of Wolff-Parkinson-White (WPW) syndrome, as evidenced 'Della wave· due 10
by the wide QRS complexes associated with relatively short PR intervals and slurring of the earty conclucltOn through
3 Initial parts of the QRS complexes. As illustrated in this diagram, an accessory conduction aooeSSOfY pathway

4 pathway in WPW leads to pre-excitation of the ventricle. The slurred upstroke of the QRS
complex Is known as a delta wave . WPW syndrome is a cause of paroxysma l
5 supraventricular tachycardia, which may lead to the symptoms experienced by this patient
as well as normal or elevated heart rate, as the name implies.
6 St pravenl cular tachycardia Electrocardiography QRS com pie>' Heart rate Ventllcle ,aeart1
0 7 Tachycardia Accessory pathway WPW syndrome Pre-excitation syndrome Delta wave
Electrical conduction system of the heart
·8
.9
• 10
• 11

• 12
• 13 A is no t co rrect. 60/o chose this.
0 14 Atrial fibrillation is seen in up to a third of people with Wolff-Parkinson-White (WPW) syndrome. This fibrillation appea rs to arise independent of the
accessory pathway of WPW. Classic ECG findings include irrregularly Irregular rhythm and absent P waves, neither of which are seen in this ECG. In
• 15 addition, the QRS complex is not widened in atrial fibrillation .
Atrial fibrillation QRS complex Electrocardiography P wave {electrocardiography) Wolff-Parkinson-White syndrome Accessory pathway Atrium {heart)
0
16
C is not co rrect . 170/o ch ose this.
0
17
Atrioventricular nodal defects are responsible for first-a nd second-degree heart block. The ECG would show an increa se in the PR interval.
. 18 Heart block Electrocardiography

• 19 D i s n ot correct. 160/o ch ose this.


• 20 Bundle-branch block would lead to a prolonged QRS complex because of abnormal depolarization (in a left bundle branch block the right ventricle Is
activated before the left ventricle; in a right bundle branch block, depolarization of the right ventricle is delayed). I t would not account for the
• 21 tachyarrhythmia .

a
Lock
s
Suspend
8
End Bl ock
Item: 6 of 25 ~. I • M k <:] t> al ~· ~
QIO: 1011 .l. ar Previous Next lab 'Vfl1ues Notes Calculator

1 Atrioventricular nodal defects are responsible for first-and second-degree heart block. The ECG would show an increa se in the PR interval.
Heart block Electrocardiography
2

3 D is no t co rrect. 16 % cho se this.


Bundle-branch block would lea d to a prolonged QRS complex because of abnormal depolarization (in a left bundle branch block the right ventricle is
4 activat ed before the left ventricle; in a right bundle branch block, depolarization of the right ventricle is delayed). It would not account for the
t achyarrhythmia.
5
Right bundle branch block QRS complex left bundle branch block Tachycardia Ventricle (heart) Cardiac arrhythmia Depolarization Bundle branch block
6 Bundle branches
.7 E is no t co rrect. 10 % cho se this.
•8 A His- Purkinje defect is represented on ECG as occasionally absent QRS complexes associat ed with constant PR and RR intervals between conducted
bea ts. This is commonly known as Mobitz type II, or second-degree heart block.
•9 Heart block Electrocardiography QRS complex
• 10

· 11
Botto m Li ne:
• 12 In Wolff-Parkinson-White syndrome, an accessory conduction pathway from atria to ventricle (known as the bundle of Kent), bypasses the
• 13 atrioventricular node .
Atrioventricular node Wolff-Parkinson-White syndrome Atrium (heart) Ventricle (heart) Accessory pathway Bundle of Kent
• 14
• 15
• 16
I ill ;fi 1!1 I•J f o r yea r :[ 2017 ..
FI RST AI D FA CTS
• 17

• 18
FA17 p 283.3
• 19
• 20
Wolff-Parkinson-White Most common type of ,·entrieular pre-
syndrome excitation syndrome. Abnormal fast accessory
• 21
conduction oathwav from atria to ventricle
6
lock
s
Suspend
0
End Block
Item: 6 of 25 ~. I • M k <:] t> al ~· ~
QIO: 1011 .l. ar Previous Next lab 'Vfl1ues Notes Calculator

1 Atrioventricular nodal defects are responsible for first-and second-degree heart block. The ECG would show an increa se in the PR interval.
Heart block Electrocardiography
2

3 D is no t co rrect. 16 % cho se this.


Bundle-branch block would lea d to a prolonged QRS complex because of abnormal depolarization (in a left bundle branch block the right ventricle is
4 activat ed before the left ventricle; in a right bundle branch block, depolarization of the right ventricle is delayed). It would not account for the
t achyarrhythmia.
5
Right bundle branch block QRS complex left bundle branch block Tachycardia Ventricle (heart) Cardiac arrhythmia Depolarization Bundle branch block
6 Bundle branches
.7 E is no t co rrect. 10 % cho se this.
•8 A His- Purkinje defect is represented on ECG as occasionally absent QRS complexes associat ed with constant PR and RR intervals between conducted
bea ts. This is commonly known as Mobitz type II, or second-degree heart block.
•9 Heart block Electrocardiography QRS complex
• 10

· 11
Botto m Li ne:
• 12 In Wolff-Parkinson-White syndrome, an accessory conduction pathway from atria to ventricle (known as the bundle of Kent), bypasses the
• 13 atrioventricular node .
Atrioventricular node Wolff-Parkinson-White syndrome Atrium (heart) Ventricle (heart) Accessory pathway Bundle of Kent
• 14
• 15
• 16
I ill ;fi 1!1 I•J f o r yea r :[ 2017 ..
FI RST AI D FA CTS
• 17

• 18
FA17 p 283.3
• 19
• 20
Wolff-Parkinson-White Most common type of ,·entrieular pre-
syndrome excitation syndrome. Abnormal fast accessory
• 21
conduction oathwav from atria to ventricle
6
lock
s
Suspend
0
End Block
Item: 6 of 25 ~. I • M k <:] t> al ~· ~
QIO: 1011 .l. ar Previous Next lab 'Vfl1ues Notes Calculator

1
>-------< Shortened PR Interval
Notmal PR 1nterva1
2

3
FA17p284.1
4
ECG tracings
5
RHYTHM DESCRIPTION EXAMPLE
6 Atrial fibrillation Chaotic and erratic baseline with no discrete P waves in between RR, • RR, • RR, • RR,
.7 irregularly spaced QRS complexes. Irregularly irregular .----.
•8 heartbeat. Most common risk factors include hypertension and
•9 coronary artery disease (CAD). Can lead to thromboembolic
Irregular baselme {absent P waves)
e,·cnts, particularly stroke.
• 10
Treatment includes anticoagulation, rate control, rhythm control,
· 11
and/or cardioversion .
• 12
Atrial flutter A rapid succession of identical, back-to-back atrial depolarization RR, = RR2 = RR,
• 13 waves. The identical appearance accounts for the "sawtooth"
• 14 appearance of the Autter wm·es.
• 15 Treat like atrial fibrillatio n. Defi nitive treatment is catheter
4:1sawtooth pattern
• 16
ablation. Ill

• 17 Ventricular A completely erratic rhythm with no identifiable waves. Fatal


fibrillation arrhythmia without immediate CPR and defibrillat ion.
• 18
• 19
No discernible rhythm
• 20
• 21
AV block

6
lock
s
Suspend
0
End Block
Item: 7 of 25 ~ 1 • M k -<:J 1>- Jil ~· !:';-~
QIO: 10 22 ..L ar Pre v ious Next Lab~lues Notes Calcula t o r
& &
1
A 54-year-old male presents to a suburban em ergency department with chest pain that started 4 days ago. He says t hat the pain is substernal and
2 feels like "a heavy pressure on my chest ." Past medical history is significant for hypercholesterolemia, hypertension, and diabetes. An ECG was
performed.
3
4 What test should be ordered to confirm his diagnosis?
5
6 A. Aspartate aminotransferase
0 7
B. Brain natriuretic peptide
o8
C. Creatine kinase-myocardial -bound fraction
.9
• 10 D. Lactate dehydrogenase isoenzyme 1

• 11 E. Myoglobin
• 12
F. Troponin
• 13
0 14
• 15
0
16
0
17
• 18
• 19
• 20
• 21

a
Lock
s
Suspend
8
End Bl ock
Item: 7 of 25 ~ 1 • M k -<:J 1>- Jil ~· !:';-~
QIO: 10 22 ..L ar Pre v ious Next Lab~lues Notes Calcula t o r
& &
1
2
3 Th e cor r ect an sw er i s F. 7 4% ch ose this.
As shown in the graph, cardiac troponin I levels become elevated in the first 4 hours after a
4 myocardial infarction (MI) and remain elevated for 7-1 0 days. It is the most specific protein
-;;;
50 ~
5 marker for MI. Troponins are also used to diagnose reinfarction, although uptrending levels E
rather than absolute elevations are used in these instances. (5
c
6 Myoca doa 1fa ction TNNIJ Troponin I Protein Troponin Infarction

7 2 !Of
.s
~

·8 Cl.l
c.
c.
:::>
.9 oglobin
• 10 ( \ C- B
• 11 I
I
AST__ LDH
, "-''-....,_ Norma
2 I
v - -- - . . . ------ \
1
• 12
I r
..
~--..... -
1 2 3 4
---
- ····· -- .. :-:~-..._.......- - - - - - - - -

5 6
----~-~r--
7 8
• 13
Daysafter Ml onset
• 14
• 15 A is not co rrect. 10/o chose this .
Aspartate aminotransferase does become elevated after myocardial Infarction, but Is not useful for diagnosis in the acute setting because It Increases
• 16 gradually over the first 2 days and then slowly declines. In addition, It Is not specific for cardiac cell damage.
• 17 Aspartate transaminase Myocardial infarction Aspartic acid Transaminase

• 18 B i s n ot co rrect. 4 0/o ch ose this .


Brain natriuretic peptide (BNP) is released by the ventricles in response to stretch as a result of volume overload, as occurs in congestive heart failure .
• 19 Similarly, atrial natriuretic peptide (ANP) is released from the myocardial cells of the atria in response to volume expansion and increased wall stress. The
• 20 plasma concentrations of both hormones are elevated in patients with heart failure. Elevations are sometimes but not always seen in cases of myocardial
Infarction (MI), more often in those causing severe dysfunction and volume overload. Consequently, it is not as specific as troponins and not used to
• 21 definitively diagnose MI.

a
Lock
s
Suspend
8
End Bl ock
Item: 7 of 25 ~. I • M k <:] t> al ~· ~
QIO: 1022 .l. ar Previous Next lab 'Vfl1 ues Notes Calculator
• •' • • ' I •
1

2 A is no t co rrect. 1% cho se this.


Aspartat e aminotransferase does become elevat ed after myocardial infarction, but is not useful for diagnosis in the acute setting because it increa ses
3 gradually over the first 2 days and then slowly declines. In addition, it is not specific for cardiac cell damage.
4 Aspartate transaminase Myocardial infarction Aspartic acid Transaminase

5 B is no t co rrect. 4 % cho se this.


Brain natriuretic peptide ( BN P) is relea sed by the ventricles in response to stretch as a result of volume overload, as occurs in congestive heart failure.
6 Similarly, atrial natriuretic peptide (AN P) is relea sed from the myocardial cells of the atria in response to volume expansion and increa sed wall stress. The
7 plasma concentrations of both hormones are elevated in patients with heart failure. Elevations are sometimes but not always seen in cases of myocardial
infarction (MI ), more often in those causing severe dysfunction and volume overload. Consequently, it is not as specific as troponins and not used to
•8 definitively diagnose MI.
Brain natriuretic peptide Atrial natriuretic peptide Myocardial infarction Congestive heart failure Heart failure Atrium (heart) Ventricle (heart) Blood plasma Peptide
•9
Brain Ventricular system
• 10
c is no t co rrect. 16 % cho se this.
· 11 Crea tine kinase- myocardial-bound (CK-MB) is an enzyme found in the myocardium but also in skelet al muscle. CK-MB fraction levels rise 6 - 12 hours after
• 12 a myocardial infarction and peak within the first 24 hours. Levels return to normal within 48 hours.
Myocardial infarction Enzyme Cardiac muscle Creatine kinase Skeletal muscle CPK-MB test Creatine Muscle
• 13
D is no t co rrect. 3 % cho se this •
• 14 Lactat e dehydrogenase isoenzyme 1 is found in cardiac cells, erythrocytes, and brain cells. Increa sed levels of isoenzyme 1 relative to lactat e
• 15 dehydrogenase isoenzyme 2 is an older; less effective method to diagnose myocardial infarction . It is the last cardiac enzyme to become significantly
elevat ed, on approximat ely day 2 after a myocardial infarction and remains elevat ed for up to 7 days .
• 16 Myocardial infarction Isozyme lactate dehydrogenase Enzyme lactic acid Red blood cell Brain Infarction

• 17 E is no t co rrect. 2 % cho se this.


• 18 Myoglobin typically rises and falls within 6 hours after a myocardial infarction (MI ). It, like aspartat e aminotransferase, is nonspecific, as it is also found in
skelet al muscle throughout the body. It is not typically used clinically in the diagnosis of MI.
• 19 Myoglobin Aspartate transaminase Myocardial infarction Skeletal muscle Aspartic acid Transaminase

• 20
• 21 Bo tto m Line:

6
lock
s
Suspend
0
End Block
Item: 7 of 25 ~. I • M k <:] t> al ~· ~
QIO: 1022 .l. ar Previous Next lab 'Vfl1ues Notes Calculator

1 • E is not correct. 2 % chose this .


Myoglobin typically rises and falls within 6 hours after a myocardial infarction (MI). It, like aspartate aminotransferase, is nonspecific, as it is also found in
2 skeletal muscle throughout the body. It is not typically used clinically in the diagnosis of MI.
Myoglobin Aspartate transaminase Myocardial infarction Skeletal muscle Aspartic acid Transaminase
3
4
5 Bottom Line:
6 Troponin is the first and most specific marker of myocardial infarction, remaining elevated from 4 hours to 7 - 10 days after the injury. CK -MB is
particularly useful for determining whether a reinfarction has occurred within days of the primary event.
7 Myocardial infarction Creatine kinase Troponin CPK-MB test Infarction

•8
•9
0 10 I iii I;fi 1!1 I•J for year:l 2017 ..
FI RST AID FAC TS
o ll
• 12 FA17 p295.1
0
13 Diagnosis of In the first 6 hours, ECC is the gold standard.
• 14
0

• 16
15
myocardial infarction Cardiac troponin I rises after 4 hours (peaks
at 24 hr) and is t for 7-10 days; more specific
than other protein markers.
!st
:§ 10
CK-MB rises after 6-12 hours (peaks at ~
Q)
0 17 a.
a.
• 18
16-24 hr) and is predominantly found
in myoca rdium but can also be released
0
"'
sl
::::>

~
Q)

0 19 from skeletal muscle. Useful in diagnosing ..,


a.
Normal
:; 2
0 20 reinfarction following acute Ml because levels ~ 1 '
return to normal after 48 hours. 1 2 3 4 5 6 7 8
0
21 Days after Ml onset •
T "A A"T I 1 o

6
lock
s
Suspend
0
End Block
Item: 7 of 25 ~ 1 • M k -<:J 1>- Jil ~· !:';-~
QIO: 10 22 ..L ar Pre v ious Next Lab~lues Notes Calcula t o r
&
• &
1
ECC changes can include ST elevation
2 (STE ll, transmural infarct), ST depression
3 (NSTEMI, subendocardial infarct),
4 hr peracute (peaked) T waves, T-wavc
5 im·ersion, new left bundle branch block, and
pathologic Q waves or poor R wa\ e progression
6
(e,·olving or old transmural infa rct).
7
·8
FA17 p 294.1
.9
• 10
Evolution of Commonly occluded coronaq arteries: I.AJ) > RCA > circumAex.
myocardial infarction Symptoms: diaphoresis, nausea, \ Omiting, sc, ere retrosternal pain, pain in left arm and/or ja\\ ,
• 11
shortness of breath, fatigue .
• 12
TIME GROSS LIGHTMICROSCOPE COMPLICATIONS
• 13
0- 24 hr I one F:nrly coagulative necrosis, Ventricular arrhythmia, I IF',
• 14
release of necrotic cell cardiogcnic shock.
• 15 contents into blood; edema,
• 16 hemorrhage, wavy fibers.
Occluded eutroph ils appear.
• 17
Rcpcrfttsion injury,
• 18 associated "it h generation
• 19 of free radica Is, leads to
• 20 ~~ Dark moltbng•
hr percontraction of myofibril s
pale wrth through t free calcium in Aux.
• 21 tecrazolium
• """'
a
Lock
s
Suspend
8
End Bl ock
Item: 8 of 25 ~ 1 • M k -<:J 1>- Jil ~· !:';-~
QIO: 1036 ..L ar Pre v ious Next Lab~lues Notes Calcula t o r
& &
1
A 32-year-old male is brought into the em ergency room in an unresponsive state. He Is accompanied by an friend who reports that the patient had
2 used heroin a few hours earlier and then "fell asleep and wouldn't wake up." Physical exam reveals a respiratory rate of 4 breaths per minute. An
arterial blood gas is ordered.
3
4 Which of the following arterial blood gas (ABG) results would you expect?
5
6 A. pH of 7.2, Paco 2 of 70, and a HC03. of 27
7
B. pH of 7.3, Paco2 of 20, and a HC03. of 10
·8
c. pH of 7.3, Paco 2 of 70, and a Hco3· of 34.5
.9
• 10 D. pH of 7.6, Paco2 of 20, and a HC03. of 20

• 11
• 12
• 13
• 14
• 15
• 16
• 17
• 18
• 19
• 20
• 21

a
Lock
s
Suspend
8
End Bl ock
Item: 8 of 25 ~ 1 • M k -<:J 1>- Jil ~· !:';-~
QIO: 1036 ..L ar Previous Next Lab~lues Notes Calculat or

3
The correct answ er i s A. 610/o chose this.
4
This ABG reflects an acute respiratory acidosis which could occur in the setting of heroin toxicity. In acute respiratory acidosis, Paco 2 is elevated and an
5 acidemia (pH < 7.36) is present because the kidneys have not had sufficient time to compensate (Hco 3 • is only minimally elevated). In acute respiratory
acidosis, HCo3· will typically rise 1 m Eq/ L for each 10 mm Hg rise in Paco 2.
6
Rate of ventilation is controlled primarily by centra l chemoreceptors sensitive to pH. Carbon dioxide can diffuse across the blood -brain barrier into the
7 cerebrospinal fluid where it combines with water to become bicarbonate and protons. This increase in addity lowers the pH and results in
hyperventilation. Accordingly, the body's appropriate response to acidosis Is to hyperventilate. In heroin toxidty, the body loses its ability to appropriately
8
ventilate and acute respiratory addosis results.
.9
Respiratory acidosis Cerebrospinal fluid Hyperventilation Carbon dioxide PH Bicarbonate Acidosis Ventilation {physiology) Blood-brain barrier Heroin Acid
• 10 Chemoreceptor Kidney Central chemoreceptors Proton Toxicity
• 11 B is not co rrect. 7 % chose this.
• 12 This ABG reflects a mixed acid base disorder. In this case, a metabolic acidosis and concomitant respiratory alkalosis is present. This ABG could occur In
the setting of salycylate toxicity, for example .
• 13 Respiratory alkalosis Metabolic acidosis Acidosis Alkalosis Toxicity Metabolism
• 14 C is not co rrect. 270/o chose this.
• 15 This ABG reflects a chronic respiratory acidosis which could occur in the setting of chronic obstructive pulmonary disea se (COPD). In a chronic respiratory
acidosis, Paco2 is elevated; however; p H is nea r normal because the kidneys have had sufficient time to compensat e with elevated bicarbonate . In chronic
• 16 respiratory acidosis, HCo 3 • will typically rise 3.5 m Eq/L for each 10 mm Hg rise In Paco>.
• 17 Respiratory acidosis Chronic obstructive pulmonary disease PH Acidosis Bicarbonate Respiratory disease Kidney

. 18 D i s n ot co rrect. solo chose this .


This ABG reflects a respiratory alkalosis. This could occur in the setting of hyperventilation from a panic attack .
• 19 Respiratory alkalosis Hyperventilation Panic attack Alkalosis
• 20
• 21
• Bottom Line:

a
Lock
s
Suspend
8
End Block
Item: 8 of 25 ~. I • M k <:] t> al ~· ~
QIO: 1036 .l. ar Previous Next lab 'Vfl1ues Notes Calculator
I' y I' y y y
1 •
D is not correct. 5 % chose this .
2
This ABG reflects a respiratory alkalosis. This could occur in the setting of hyperventilation from a panic attack.
3 Respiratory alkalosis Hyperventilation Panic attack Alkalosis

4
5 Bottom Line:
6 Heroin overdose leads to depression of the central respiratory drive. An increase in carbon dioxide due to hypoventilation causes an acute respiratory
acidosis. You must be able to interpret ABGs and correlate them with clinical scenarios for step 1.
7 Respiratory acidosis Carbon dioxide Hypoventilation Heroin Acidosis Opioid overdose Control of ventilation Major depressive disorder Drug overdose
8
•9
• 10 I iii I;fi 1!1 I•J for year:l 2017 ..
FI RST AID FA CTS
· 11
• 12
FA17 p 285.3
• 13
Baroreceptors and chemoreceptors Receptors:
• 14
AFFERENT EFFERENT
• Aortic arch transmits via vagus nerve to sol itary nucleus of
• 15 medulla (responds to ! and t in BP).
• 16 • Ctuotid sinus (dilated region at carotid hi furcation) transmits via
• 17 glossopharyngeal nerve to solitary nucleus of medulla (responds
/" Sympathetic to ! and t in BP).
• 18 chain
• 19
Baroreceptors:
• Hypotension-! arterial pressure .... l stretch .... l afferent
• 20
Para.sympathetic baroreceptor firing .... t efferent sympathetic firin g and
• 21 vagus nerve
• I ! efferent parasympathetic stimulation .... vasoconstriction,

6
lock
s
Suspend
0
End Block
Item: 8 of 25 ~. I • M k <:] t> al ~· ~
QIO: 1036 .l. ar Previous Next lab 'Vfl1ues Notes Calculator

1
el g - - I 2 p- I

• Central-are stimulated by changes in pH and Pco2 of brain


2
interstitial Auid, which in turn are influenced by arterial C02•
3 Do not di rectly respond to Po2•
4
5 FA17 p 286.2
6
Autoregulation How blood Aow to an organ remains constant over a wide range of perfusion pressures.
7
ORGAN FACTORSDETERMINING AUTOREGULATION
8
Heart Local metabolites (vasodilatory): adenosine, ote: the pulmonary vasculature is unique in
•9
NO, C02, ! 0 2 that hypoxia causes vasoconstriction so that
• 10 only well-ve ntilated areas are perfused. In
Brain Loca l metabol ites (vasodilatory): C02 (pH)
· 11 other organs, hypoxia causes vasodilation.
Kidneys Myogenic and tubuloglomerular feedback
• 12
Lungs Hypoxia causes vasoconstriction
• 13
Skeletal muscle Loca 1 metabol ites during exercise: lactate, C 0 2, 11+, Adenosine, Lactate, K+ (C H ALK)
• 14
adenosine, K+, H+, C0 2
• 15
At rest: sympathetic tone
• 16
Skin Sympathetic stimulation most important
• 17 mechanism for te mperature control
• 18
• 19 FA11 p 561 .1
• 20 Acid-base physiology
• 21 pH Pco [HCO I COMPENSATORYRESPONSE

6
lock
s
Suspend
0
End Block
Item: 9 of 25 ~ 1 • M k -<:J 1>- Jil ~· !:';-~
QIO: 3211 ..L ar Pre v ious Next Lab~lues Notes Calcula t o r
& &
1
A 45-year-old man presents to his physician complaining of edema in his lower extremities. Physical examination revea ls mild-to-moderate pitting
2 edema up to the level of the knee bilaterally, and a new grade 2 systolic ejection murmur that radiates to the carotids. The physician refers the
patient to a cardiologist, who performs a thorough work-up, which shows:
3
R-R Interval: 0.8 sec
4 Mean pulmonary artery pressure: 15 mm Hg
Right atrial pressure: 2 mm Hg
5 Left atrial pressure: 5 mm Hg
6 Mean arterial pressure (aorta): 95 mm Hg
Oxygen consumption (whole body): 250 mljmin
7 Oxygen content of femoral arterial blood: 0.20 ml O:Jml blood
Oxygen content of pulmonary arterial blood: 0.15 ml O:Jml blood
8
.9
What Is the stroke volume for this patient?
• 10
• 11
A. 8.33 ml
• 12
• 13
B. 10.4 ml

• 14 c. 66.7 ml

• 15
o. 83.3 ml
• 16
E. 104.2 ml
• 17
• 18 F• 667 ml

• 19 G. 833 ml
• 20
• 21

a
Lock
s
Suspend
8
End Bl ock
Item: 9 of 25 ~ 1 • M k -<:J 1>- Jil ~· !:';-~
QIO: 3211 ..L ar Pre v ious Next Lab~lues Notes Calcula t o r

1
&
G. 833 mL &

3
4
The correct a nswer is c . 53% chose t his.
5 The end-diastolic volume (EDV) and end -systolic volume (ESV) of t he left ventricle are not given and t hus t he stroke volume (SV) cannot be calculated
6 directly (SV =
EDV - ESV). However, sv ca n be ca lcul ated by remembering that cardiac output (CO) = sv x heart rate (HR), and by recognizing that the
CO and HR can be calculated from t he inform ation give n. co can be calculated using the Fick principl e, which states t hat t he rate of oxygen utilization by
7 the body Is equal to the co tim es the diffe re nce bet ween the o xygen content of the systemic arterial blood (fe mora l a rte ry) a nd the oxygen content of
the systemic venous blood (pulmon ary a rte ry). Th e equ ation can be rearranged to solve for co as follows:
8
CO (ml blood / min) = oxygen consumption by the whole body (ml/ min) ..;. (arterial oxygen content [ml oxygen / ml blood] - venou s
9 oxyg en c ont ent (ml oxygen / ml blood]) .
• 10 Thus, In this case,
• 11 CO = 25 0 ml o xygen/min ..;. (0.20 ml oxygen/ml blood - 0.15 ml o xygen/ml blood) = 5000 ml/min

• 12 Next, we must calculate the HR. The R-R int erval (illustrated in the diagram below) tells us how long each beat takes (seconds/ beat). The Inverse of the
R-R Interval gives us beats/second, which can be mult iplied by 60 (60 seconds/minute) to give HR (beats/min ). In t his case,
• 13
RR = 0.8 sec/beat. 1/RR = 1.25 beats/sec. HR = 1.25 beats/sec x 60 sec/mi n = 75 beats/min
• 14
sv can then be calculated from the co and HR :
• 15
sv ( m l blood/beat) = co (ml blood/min) I HR (beats/min) = 5000 m l / min ..;. 75 bea t s/ min = 66.7 ml/beat
• 16
Stroke volume Femoral artery Cardiac output Fick principle Pulmonary artery Ventricle (heart) End-systolic volume Electrocardiography Heart rate
• 17 End- diastolic volume Stroke Systole Venous blood Oxygen

~------~R-~R~I~nt~e~N~al~-----~
. 18
• 19
• 20
• 21
• ---A~ T ~~
a
Lock
s
Suspend
8
End Bl ock
Item: 9 of 25 ~ 1 • M k -<:J 1>- Jil ~· !:';-~
QIO: 3211 ..L ar Previous Next Lab~lues Notes Calculat or
&
·····- - - - -- - - --- ... - &
1
sv can then be calculated from the co and HR:
2
SV ( ml b l ood/b ea t ) = CO ( ml blood/ min) I HR ( b ea t s/min) = 5 000 ml/ min .;. 75 b ea t s/min = 66.7 ml/ b ea t
3
Stroke volume Femoral artery Cardiac output Fick principle Pulmonary artery Ventr cle (heart) End-systolic volume Electrocardiography Heart rate
4 End-diastolic volume Stroke Systole Venous blood Oxygen
5 ~------~R~~I~nt~eN~al_______~
6
7
8
9 I I
as
• 10 Image courtesy of Ted Burke
• 11
A is n ot co rrect. 6 0/o ch ose this •
• 12 The correct answer is 66.7 ml; 8.3 ml is too small to be the volume of blood ejected from the ventricle per beat.
Ventricle (heart)
• 13
• 14 6 is not co rrect. 90/o chose this •
The correct answer is 66.7 ml, and 10.4 ml is too small to be the volume of blood ejected from the ventricle per bea t
• 15 ventricle (heart)
• 16 D is no t co rrect . 1 60/o ch ose this.
• 17 The correct answer is 66.7 ml; 83.3 ml is too large to be the correct stroke volume .
Stroke volume Stroke Milliliter Utre
. 18
E i s n ot co rrect. 9 0/o ch ose this .
• 19
If the R-R Interval (0.8 sec) were used for the HR, instead of the inverse of the R-R interval (1.25 beats/ sec), this would mistakenly give an HR of 48
• 20 beats/ min instead of 75 beats/ min. If 48 beats/ min were then used In the SV calculations, this would give an sv of 104.2 ml, instead of the correct
answer, which is 66.7 ml.
• 21 Elect• ocard•ography M iter

a
Lock
s
Suspend
8
End Block
Item: 9 of 25 ~. I • M k <:] t> al ~· ~
QIO: 3211 .l. ar Previous Next Lab 'Vfll ues Notes Calculator

1
.. . g
Stroke volume Stroke Milliliter litre
2
E is no t co rrect. 9 % cho se this.
3 If the R-R interval (0 .8 sec) were used for the HR, instea d of the inverse of the R-R interval ( 1.25 bea ts/ sec), this would mistakenly give an HR of 48
bea ts/ min instea d of 75 bea ts/ min. If 48 bea ts/ min were then used in the SV calculations, this would give an SV of 104.2 ml, instea d of the correct
4
answer; which is 66.7 ml.
5 Electrocardiography Milliliter

6 F is no t co rrect. 6 % cho se this.


The correct answer is 66.7 ml, and thus 667 ml is off by a factor of 10 . This answer can be eliminat ed because it is too large a value for the volume of
7
blood ej ected from the ventricle per bea t
8 Ventricle (heart)

9 G is no t co rrect. 1 % cho se this.


The correct answer is 66.7 ml, and 833 ml is too large to be the volume of blood ej ected from the ventricle per bea t .
0 10
Ventricle (heart)
o ll
0
12
Botto m Li ne:
0
13
It is important to memorize the key equations for calculating cardiac output and stroke volume, as follows:
0 14
o co = sv x HR; co = mean arterial pressure 1 total peripheral resistance
0 15 o CO = (rat e of oxygen consumption) I (arterial oxygen - venous oxygen)
o SV = EDV - ESV
0
16
Vascular resistance Stroke volume Mean arterial pressure Cardiac output Stroke Oxygen Vein
0 17

0
18

0 19
lijj ;fi IJ l•l f o r yea r :l 2017 ..
0 20 FI RST AI D FA CTS

0
21
• -·.. - - ~ -,.. ..
6
lock Suspend
s 0
End Block
Item: 9 of 25 ~ 1 • M k -<:J 1>- Jil ~· !:';-~
QIO: 3211 ..L ar Pre v ious Next Lab~lues Notes Calcula t o r

2
3 141;fiiJI•J for y ear: 2017 ...
FIRST o\ID FIICTS
4
5
FA17 p 272.1
6
Cardiac output CO= stroke volume (SV) x heart rate (HR) During the early stages of exercise, CO is
7
Fick principle: maintained b) t HR and t SV. During the late
8 stages of exercise, CO is maintained by t HR
CO= rate of07 consumption
9 only (SV plateaus).
arterial 0 1 content -venous 0 2 content
• 10 Diastole is preferentially shortened with t IIR;
Nlean arterial pressure (M P) =CO x tol<ll less filling Lime ..... l CO (eg, ventricular
• 11
peripheral resistance (TPR) tacll)'Cardia).
• 12
• 13
NlAP = 2/, diastolic pressure+ Xsystolic pressure
• 14 Pulse pressure= systolic pressure - diastolic pressure t pulse pressure in hyperthyroidism, aort ic
• 15 Pulse pressure is proportional to S , inversely regurgitation, aortic stiffening (isolated systolic
proportional to arterial compliance. hypertension in elderly), obstructive sleep
• 16
apnea (t sympathetic tone), exercise (transient).
• 17 S =end-diastolic volume (ED ) -end-systolic
l pulse pressure in aortic stenosis, cardiogcnic
• 18
\·olume (ESV)
shock, cardiac tamponade, advanced heart
• 19 fai lme {H F').
• 20
• 21 FA17 p 273.1

a
Lock Suspend
s 8
End Bl ock
Item: 10 of 25 ~ 1 • M k -<:J 1>- Jil ~· !:';-~
QIO: 10 18 ..L ar Pre v ious Next Lab~lues Notes Calcula t o r

1
&

A 56-year-old Asian man with hypercholesterolemia and type 2 diabetes mellitus comes to a physician for a check-up. It has been several years since
he has been to the doctor. His past medical history is significant for an acute Illness at the age of 9 which involved a high fever, pleuritic chest pain,
IA•A] &

2
migrating joint pain, and a pink, nonpruritic rash on his torso . His blood pressure Is 135/85 mm Hg and heart rate is 70/min. Auscultation of the heart
3 reveals a low-pitched diastolic rumble heard best at the apex.
4
5 What Is the most likely pressure change that would be seen in this patient's heart?

6
A. Decreased left atrial pressure
7
8 B. Decreased left ventricular pressure

9 c. Increased left atrial pressure


• 10
D. Increased left ventricular pressure
• 11
E. Increased right atrial pressure
• 12
• 13
• 14
• 15
• 16
• 17
• 18
• 19
• 20
• 21

a
Lock
s
Suspend
8
End Bl ock
Item: 10 of 25 ~ 1 • M k -<:J 1>- Jil ~· !:';-~
QIO: 1018 ..L ar Prev ious Next Lab~lues Notes Calculat o r

& &
1

2 Th e correct an sw er i s c. 690/o chose this.


3 The patient's history of a prior illness with fea tures of fever, pleuritic chest pain, joint pain, and rash
(probably erythema marginatum) is indicative of rheumatic fever. Rheumatic fever can evolve into
4 rheumatic heart drsease, which typically manifests with mitral valve stenosis (shown in the image). This
5 can be heard on auscultation as a high-pitched opening snap that follows 52, and a low-frequency
decrescendo diastolic murmur heard best over the apex of the heart. The Increased resistance to flow from
6 the left atrium to the left ventrlde leads to an increase in left atrial pressure.
Rhe1 mati ever Auscultation Mitral valve stenosis Ventricle lheart· Mitra valve Pie• •SY Stenosis Atrium (heart)
7
Diastole Arthralgia Apex beat Chest pain Fever Erythema Cardiovascular disease Heart murmur
8
9
10
Image courtesy of CDC/ Dr. Edwin P. Ewing,
• 11 Jr.
• 12
A is not co rrect. 40/o chose this •
• 13
Left atrial pressure would increase, not decrease, with mitral stenosis. This Is because the stenosis creates a pressure gradient between the left ventricle
• 14 and the left atrium. In mitral stenosis, this pressure gradient across the mitral valve can lead to elevated left atrial pressures which may eventually lead
to an Increase In the size of the left atrium .
• 15 Mitral valve stenosis Ventricle (heart) Stenosis Mitral valve Atrium (heart) Ventricular system

• 16 B is not co rrect . 90/o ch ose this.


• 17 Mitral stenosis would not result in an overall decrea se in left ventricular pressure; pressure will only be decrea sed relative to the left atrium .
Mitral valve stenosis Atrium (heart) Stenosis Mitral valve Ventricular system Heart
. 18
D i s n ot correct. 150/o ch ose this .
• 19
Left ventricular pressure would increase with aortic stenosis. Rheumatic heart disease most typically manifests as mitral stenosis, although aortic stenosis
• 20 Is the next most common presentation .
Left ventr cular pressure Aortic stenosis Rheumatic fever Mitral valve stenosis Stenos1s Cardiovascular disease Ventricular system Heart Rheumatism
• 21
• Rheumatoloov

a
Lock
s
Suspend
8
End Block
Item: 10of25 ~. , . M k <:] t> al ~· ~
QIO: 1018 .l. ar Previous Next lab 'Vfl1ues Notes Calculator
- I 'f i'J I I - 'J e:.: - 1 - - - - :.; - 'J I - 'J - I - 'J - 'f I :.: 1 - 1 - - - · - :.; - I - - 'J
1 Mitral valve stenosis Atrium (heart) Stenosis Mitral valve Ventricular system Heart
2
D is not correct. 15% chose this.
3 Left ventricular pressure would increa se with aortic stenosis. Rheumatic heart disea se most typically manifests as mitral stenosis, although aortic stenosis
is the next most common presentation .
4
left ventricular pressure Aortic stenosis Rheumatic fever Mitral valve stenosis Stenosis Cardiovascular disease Ventricular system Heart Rheumatism
5 Rheumatology

6 E is not correct. 3 % chose this.


7 Right atrial pressure would increa se in response to pulmonic valve stenosis or tricuspid va lve stenosis. The tricuspid valve is sometimes affected in
rheumatic heart disea se and would also cause a diastolic murmur. However; this is far less common than mitral stenosis in the setting of rheumatic heart
8 disea se. In fact, the pulmonic valve is the lea st likely to be affected in rheumatic heart disea se.
Tricuspid valve stenosis Pulmonary valve Rheumatic fever Tricuspid valve Mitral valve stenosis Stenosis Diastole Atrium (heart) Cardiovascular disease
9
Tricuspid atresia Heart murmur
10
· 11
Bottom Line:
• 12
Consider mitral stenosis in a patient with a diastolic rumbling murmur in the setting of rheumatic heart disea se. The left atrial pressure and pulmonary
• 13 pressures are often elevat ed, sometimes lea ding to atrial fibrillation as the left atrium dilat es.
Mitral valve stenosis Rheumatic fever Atrial fibrillation Atrium (heart) Stenosis Diastole Cardiovascular disease Mitral valve Fibrillation Rheumatism Heart murmur
• 14
• 15
• 16
• 17
I iii I;fi 1!1 I•J for year:l 2017 ..
FI RST AI D FA CTS

• 18
• 19 FA17 p278.1

• 20 Auscultation of the heart


• 21

6
lock
s
Suspend
0
End Block
Item: 10 of 25 ~ 1 • M k -<:J 1>- Jil ~· !:';-~
QIO: 10 18 ..L ar Pre v ious Next Lab~lues Notes Calcula t o r

1 g g p •
2

3 FA17p279.1
Heart murmurs
4
5 Systolic

6 Aortic stenosis Crescendo-decrescendo systolic ejection murmur (ejection click may be present).
LV>> aortic pressure during systole. Loudest at heart base; radiates to carotids.
7 Sl S2
'·Pulsus pan·us et tardus''-pulses are weak with a delayed peak. Can lead to
8
9
~I Syncope, \ ngina, and Dyspnea on exert ion (S.\0). lost commonly due to age-
related calcification in older patients (> 60 years old) or in younger patients with
10 early-onset calcification of bicuspid aortic ,·akc.
• 11 Mitral/tricuspid regurgitation Holosystolic, high-pitched "blowing murmur:·
• 12 Sl 52 Mitral-loudest at apex and rad iates toward axilla. IRis often due to ischemic heart
disease (post-M I), MVP, LV dilatation.
~
• 13
Tricuspid- loudest at tricuspid area. TR commonly caused by RV dilatation .
• 14
Rheumatic fever and infective endocarditis can cause either IR or TR .
• 15
Mitral valve prolapse Late systolic crescendo murmur with midsystolie click (MC; due to sudden tensing
• 16
Sl MC S2 of chordae tendineac). Most frequent vah-ular lesion. Best heard over apex. Loudest
• 17 just before S2. Usually benign. Can predispose to infective endocarditis. Can be
. 18
• 19
I LwHNI zo
caused by myxomatous degeneration (1° or to connecti\·e tissue disease such as
Marfan or Ehlers-Dan los syndrome), rheumatic fe,·er, chordae rupture.
• 20 Ventricular septal defect llolosystolic, harsh-sounding murmur. Loudest at tricuspid area.
• 21
• Sl S2

a
Lock
s
Suspend
8
End Bl ock
Item: 10 of 25 ~ 1 • M k -<:J 1>- Jil ~· !:';-~
QIO: 10 18 ..L ar Pre v ious Next Lab~lues Notes Calcula t o r

1
&
Sl 52 &

2
3
4
5 FA17 p 300.1
6 Rheumatic fever A consequence of pharyngeal infection with J• '\ES (major criteria}:
7 group A P-hemolytic streptococci. Late Joint (migratory polrarthritis)
8 sequelae include rheumatic heart di ease, • (carditis)
which affects heart valves-mitral> aortic>> l\odules in skin (subcutaneous)
9
tricuspid (high-pressure valves affected most). Erythema marginatum
10
Early lesion is mitral valve regurgitation; Sydenham chorea
• 11 late lesion is mitral stenosis. Associated
• 12 with Aschoff bodies (granuloma " ilh giant
• 13 cells [blue arrows in rJ]), Anilschkow cells
• 14
(enlarged macrophages with ovoid, wavy,
rod-like nucleus [red arrow in rJ 1), t anti-
• 15
streptolysin 0 (A$0) tilers .
• 16 Immune mediated (type II hypersensit ivity};
• 17 not a direct effect of bacteria. Ant·ibodies
• 18 toM protein cross-react with self antigens
• 19 (molecular mimicry).
Treatment/prophylaxis: penicillin .
• 20
• 21

a
Lock
s
Suspend
8
End Bl ock
Item: 11 of 25 ~ 1 • M k -<:J 1>- Jil ~· !:';-~
QIO: 5 0 84 ..L ar Pre v ious Next Lab~lues Notes Calcula t o r
& &
1
A 23-year-old intravenous heroin user is brought to the emergency department by ambulance after being found unresponsive. The patient is
2 resuscitated with naloxone and is admitted to the hospital, where he has a fever of 39°C (102.2°F). Upon examination, a "blowing" systolic heart
murmur Is heard best at the lower sternal border. The murmur varies with respiration, becoming louder with inspiration. Additionally, the examiner
3 notes a prominent jugular venous pulse.
4
5 Which of the following va lvular abnormalities is most consistent with the murmur In this patient?

6
A. Aort1c regurgitation
7
8 B. Aortic stenosis
9 c. Mitral regurgitation
10
o. Mitral stenosis
• 11
• 12
E. Tricuspid regurgitation

• 13 F• Tricuspid stenosis

• 14
• 15
• 16
• 17
• 18
• 19
• 20
• 21

a
Lock
s
Suspend
8
End Bl ock
Item: 11 of 25 ~ 1 • M k -<:J 1>- Jil ~· !:';-~
QIO: 5084 ..L ar Prev ious Next Lab~lues Notes Calculat o r

2
3
The correct an swer is E. 670/o ch ose this.
4
This patient has a systolic "blowing " murmur, which is loudest at the lower sterna l border and increases
5 with Inspiration. This is most consistent with tricuspid regurgitation. While tricuspid regurgitation may be
caused by right ventricular dilatation, t he tricuspid valve is also the most common s1te of Infective
6 endocarditis 1n intravenous drug users.

7 The arrow in the echocardiograph indicates a vegetation on the t ricuspid va lve.

8 Tricuspod v, lve Infective endocarditis Endocarditis Echocardiography Systole •ric ospid insufficiency

9 Regurgitation (circulation) Sternum Drug injection Intravenous therapy Tricuspid atresia Heart murmur

10
11
• 12 Images copyright © 2004 Koya eta/;
licensee BioMed Central Ltd.
• 13
• 14 A is not co rrect. JO/o chose t his.
• 15 The murmur of aortic regurgitation is a high -pitched, "blowing" diastolic murmur. It is associated with a wide pulse pressure and is often caused by aortic
root dilation, bicuspid ao1tic va lve, endocarditis, and rheumatic fever, which may all cause blood to flow backwards through the ao1tic va lve during
• 16 diastole. Severe aortic regurgitation is classica lly associated with head bobbing.
Rheumatic fever Aortic insufficiency Endocarditis Bicuspid aortic valve Diastole Aortic valve Pulse pressure Regurgitation (circulation) Ascending aorta Aorta
• 17
Mitral valve Fever Heart murmur Vasodilation
• 18
B i s n ot correct. JOfo ch ose this .
• 19
The murmur of aortic stenosis is a crescendo-decrescendo ejection murmur (sometimes with an ejection click), which radiates to the carotid arteries. In
• 20 older patients, it is due to valve cacification and it is important to suspect early-onset calficication of a bicuspid aortic valve in younger patients. Aortic
stenosis Is associated with a "pulsus parvus et tardus" or slow, weak pulse. This is a systolic murmur and left ventricular pressure will be much higher than
• 21 the aortic pressure; the reduction in systolic blood flow can lead patients to present with syncope, angina, and dyspnea .

a
Lock
s
Suspend
8
End Bl ock
Item: 11 of 25 ~ 1 • M k -<:J 1>- Jil ~· !:';-~
QIO: 5 0 84 ..L ar Previous Next Lab~lues Notes Calculat or
& &
1 Mitral valve Fever Heart murmur Vasodilation
2
B i s n ot correct. 30/o ch ose this.
3 The murmur of aortic stenosis is a crescendo-decrescendo ejection murmur (sometimes with an ejection click), which radiates to the carotid arteries. In
older patients, it is due to va lve cacification and it is important to suspect early-onset calficication of a bicuspid aortic valve in younger patients. Aortic
4 stenosis Is associated with a "pulsus parvus et tardus" or slow, weak pulse. This is a systolic murmur and left ventricular pressure will be much higher than
5 the aortic pressure; the reduction in systolic blood flow can lead patients to present wrth syncope, angina, and dyspnea.
Dyspnea ICUSIJ'd aortic valve Aortic stenosis Aortic valve Common carotid artery C otid artery Syncope (medicine) Systole Stenosis Angina pectoris Artery
6 Mit a val re U!ft •entricular pressure Heart murmur Heart Blood flow
7 C i s not correct. 1 0 % chose this.
8 The murmur of mitral regurgitation is a holosystolic " blowing " murmur heard lo udest at the apex. It typically rad iates to the axill a and does not Increase
with Inspiration. Mitral regurgitation may occur in patients who have had a myocardial infarction, mitral valve prolapse, left ventricular dilation, rheumatic
9 fever, and Infective endocarditis.
10 Rheumatic fever Mitral valve prolapse Axilla Myocardial infarction Endocarditis Mitral insufficiency Infective endocarditis Mitral valve Regurgitation (circulation)
Prolapse Fever Heart Heart murmur
11
0 is no t co rrect. 4 0/o chose t his •
• 12
The murmur of mitral stenosis (often associated with rheumatic fever) Is a late diastolic murmur that follows an opening snap. This snap is due to an
• 13 abrupt halt In leaflet motion due to fusion at the leaflet tips) and left atrial pressure will be much higher than left ventricular pressure during diastole.
Chronic m itral stenosis may lead to left atrial dilat ation . The murmur becomes worse when the opening snap moves closer to the 52 heart sound .
• 14
Rheumatic fever Heart sounds Mitral valve stenosis Diastole Stenosis Rheumatology Heart murmur Fever Heart
• 15
F is not co rrect . 130/o chose this .
• 16 Tricuspid stenosis is rare . Tricuspid stenosis m anifests with a diastolic murmur, unlike this patient's systolic murmur; and the murmur increases with
Inspiration .
• 17
Tricuspid valve stenosis Stenosis Systole Diastole Tricuspid valve Systolic heart murmur Tricuspid atresia Heart murmur
. 18
• 19
Bottom Line:
• 20
Intravenous drug use is associated with tricuspid endocarditis.
• 21 Endocardit s Intravenous therapy Drug injection Tricuspid valve Recreational drug use

a
Lock
s
Suspend
8
End Block
Item: 11 of 25 ~. I • M k <:] t> al ~· ~
QIO: 5084 .l. ar Previous Next lab 'Vfl1 ues Notes Calculator

1 Tricuspid valve stenosis Stenosis Systole Diastole Tricuspid valve Systolic heart murmur Tricuspid atresia Heart murmur

3 Botto m Line:
4 Int ravenous drug use is associated wit h t ricuspid endocardit is.
Endocarditis Intravenous therapy Drug injection Tricuspid valve Recreational drug use
5
6
7
I iii I;fi 1!1 I•J fo r yea r:l 20 17 ..
8 FI RST AI D FA CTS

9
10 FA17p279.1
Heart murmurs
11

• 12
Systolic

• 13 Aortic stenosis Crescendo-decrescendo systolic ejection murmur (ejection click may be present).
LV >> aortic pressure during systole. Loudest at heart base; radiates to carotids.
• 14 Sl 52
"Pulsus pan·us et ta rdus"- pulses are weak with a delayed peak. Can lead to
• 15
• 16
~I Syncope, Angina, and Dyspnea on exertion (SAD). Most commonly due to age-
related calcification in older patients (> 60 years old) or in younger patients with
• 17 early-onset calcification of bicuspid aortic vah-e.
• 18 Mitral/tricuspid regurgitation Holosystolic, high-pitched "blowing murmur."
• 19 51 52
Mitral - loudest at apex and rad iates toward axilla. MR is often due to ischemic heart
disease (post-Ml), MVP, LV dilatation.
~
• 20
Tricuspid- loudest at tricuspid area. T R commonly caused by RV dilatation.
• 21
Rh f'nm::ttiC' ff'vf'r ::tnrl inff'l'l'i vf' f'ndoC'::trrlitis C'::t n C'::tiiSf' f'ithf'r 1\11 R or T R.

6
lock
s
Suspend
0
End Block
Item: 11 of 25 ~ 1 • M k -<:J 1>- Jil ~· !:';-~
QIO: 5 0 84 ..L ar Pre v ious Next Lab~lues Notes Calcula t o r
& &
1

2 I ,. . ""~"'
3
4
FA17 p 299.2
5
6
Bacterial endocarditis Fe,·er (most common symptom), new murmur, \1 itral ,-ah-e is most frequently invoked.
Roth spots (round '' hite spots on ret ina 'I ricuspid ,·a h-e endocarditis is associated ''it h
7
surrounded br hemorrhage }, Osler nodes I drug abuse (don't - tri" dmg~)- Associated
8 (tender raised lesions on finger or toe pads '' ith S aureus, Pseudomonas, and Candida.
9 due to immune complex deposit ion}, Jane\\'a}' Culture 8; most likely Coxiella bumetii,
10 lesions (small, painless, erythematous lesions Bartonella spp., IIACEK (1-laemophi/us,
11
on palm or sole) ~. glomerulonephritis, Aggregatibacter (formerly Actinobacillus),
septic arterial or pulmomuy emboli, splinter Cardiobacterium, Eikenella, Kingella)
• 12
• 13
hemorrhages [!] on nail bed. Multiple blood • Bacteria FRO~ l JANE •=
cultures necessary for diagnosis. Fc,·cr
• 14 • Acute- S aureus (high virulence). Roth spots
• 15 Large vegetations on previous!)' normal Osier nodes
• 16 valves (). Rapid onset. 1\ lmmur
• Subacute-viridans streptococci (low Janeway lesions
• 17
virulence). Smaller vegetations on Anemia
. 18
congenitally abnormal or diseased valves. l\ail-bed hemorrhage
• 19 Sequela of dental procedures. Gradual Fmboli
• 20 onset.
• 21 S bovis (gallolyticus) is present in colon cancer,
• r . · 1 • r· .1 • • 1

a
Lock
s
Suspend
8
End Bl ock
Item: 12 of 25 ~ 1 • M k -<:J 1>- Jil ~· !:';-~
QIO: 3885 ..L ar Pre v ious Next Lab~lues Notes Calcula t o r
& &
1
A 65-year-old man with a history of myocardial infarction presents with shortness of breath when climbing the stairs to his apartment. His feet are
2 more swollen than usual, and he recently has begun to sleep propped up In his armchair. Medications at home include omeprazole, glipizide, and
slmvastatln. His primary care physician decides to prescribe a new medication to treat the patient's symptoms.
3
4 Which of the following medications, although not absolutely contraindicated, should be used with caution due to the theoretica l concerns of masking
5 hypoglycemia?

6
A. Captopril
7
8 B. Digoxin
9 c. Hydralazine
10
o. Hydrochlorothiazide
11
• 12
E. Metoprolol

• 13
• 14
• 15
• 16
• 17
• 18
• 19
• 20
• 21

a
Lock
s
Suspend
8
End Bl ock
Item: 12 of 25 ~ 1 • M k -<:J 1>- Jil ~· !:';-~
QIO: 3885 ..L ar Previous Next Lab~lues Notes Calculat or
& &
1
2 Th e correct a n swer i s E. 570/o chose this.
3 13-blockers such as metoprolol decrease heart rate, contractility, and conduction through the atrioventricular node. They have been shown to decrease
mortality and disease progression in patients with stable systolic congestive heart failure. Patients need to be mindful t hat 13-blockers may mask the signs
4 and symptoms of hypoglycemia (eg, diaphoresis, tachycardia). This patient Is taking a sulfonylurea, glipizide, for t he treatment of diabetes. Sulfonylureas
trigger the release of insulin and therefore are associated with a risk of hypoglycemia.
5
Sulfonylu ea At1ooventrocular node Metoprolol Glipizide Hypoglycemia Heart fa lu e Pe1 s~ ation Congestive heart failure Tachycardia Diabetes mel !tus Heart rate
6 Ins n C •tract' ity Systole
7 A is not correct. 8 % chose this.
8 Captoprll Is an angiotensin-converting enzym e (ACE) inhibitor that is used for the treatment of hypertension and congestive heart fa ilure. ACE Inhibitors
reverse the effect of the angiotensin 11-induced vasoconstriction, promoting Increased myocardial blood flow and red uced system ic vascular resistance.
9 Captoprll would not interact with the patie nt's medications.
10 Captop il Heart failure Angiotensin-converting enzyme ACE inhibitor vasoconstriction Enzyme Congestive heart failure Hypertension vascular resistance
Angiotensin Enzyme inhibitor Blood vessel
11
B is not co rrect. 9% chose t his.
12
Cardiac glycosides such as digoxin are used in congestive heart failure to Increase contractility. It inhibits the sodium/potassium/ATPase pump, causing
• 13 Increased Intracellular sodium, increased intracellular calcium, and, ultimately, positive inotropy. Digoxin typically is avoided in patients with renal failure
and hypokalemia, as these potentiate the drug's effect, and in patients taking quinidine, which decreases digoxin cleara nce .
• 14 Quinidine Hypokalemia Digoxin Heart failure Congestive heart failure Cardiac glycoside Glycoside Inotrope Sodium Contractility Intracellular Calcium Kidney
• 15 C is no t co rrect. 90/o ch ose this .
• 16 Hydralazine is a va sodilator that directly induces smooth muscle relaxation . In the treatment of congestive heart failure, hydralazine is used in
combination with nitrates to reduce systemic vascular resistance. It would not interact with the patient's medications.
• 17 Hydralazine Vasodilation Heart failure Congestive heart failure vascular resistance Smooth muscle tissue Nitrate Muscle relaxant Blood vessel
• 18 D i s n ot correct. 170/o ch ose this .
• 19 Thiazide diuretics such as hydrochlorothiazide inhibit sodium chloride resorption in the distal convoluted tubules of t he kidneys. I n congestive heart
failure, diuretics are used to reduce volume overload. However, the patient Is taking glipizide for blood sugar m anagement. Thiazide diuretics can result in
• 20 hyperglycemia. Nevertheless, the consequence of t hiazide use would not lead to masking symptoms of hypoglycemia.
• 21 Hydrochlorothiazide Thiazide Sodium chloride Hypoglycemia Heart fa lure Hyperglycem•a Congestive heart failure Diuretic Glipizide Blood sugar

a
Lock
s
Suspend
8
End Block
Item: 12of25 ~. , . M k <:] t> al ~· ~
QIO: 3885 .l. ar Previous Next lab 'Vfl1ues Notes Calculator

1 Hydralazine Vasodilation Heart failure Congestive heart failure Vascular resistance Smooth muscle tissue Nitrate Muscle relaxant Blood vessel

2 D is not correct. 17% chose this.


Thiazide diuretics such as hydrochlorothiazide inhibit sodium chloride resorption in the distal convoluted tubules of the kidneys. In congestive heart
3
failure, diuretics are used to reduce volume overload. However; the patient is taking glipizide for blood sugar management. Thiazide diuretics can result in
4 hyperglycemia. Nevertheless, the consequence of thia zide use would not lead to masking symptoms of hypoglycemia.
Hydrochlorothiazide Thiazide Sodium chloride Hypoglycemia Heart failure Hyperglycemia Congestive heart failure Diuretic Glipizide Blood sugar
5
Distal convoluted tubule Sodium Kidney Chloride Nephron
6
7
Bottom Line:
8
13-Biockers such as metoprolol decrease heart rate, contractility, and conduction through the atrioventricular node. They should be used with caution in
9 patients with diabetes because of the risk of masking the symptoms of hypoglycemia.
Atrioventricular node Metoprolol Hypoglycemia Diabetes mellitus Heart rate Contractility
10
11

12
lijj ;fi IJ l•l for year:l 2017 ..
• 13 FI RST AID FAC T S

• 14

• 15 FA17p237.1

• 16 p~blockers Acebutolol, atenolol, betaxolol, bisoprolol, carvedilol, esmolol, labetalol, metoprolol, nadolol,
• 17
nebivolol, pindolol, propranolol, timolol.
APPLICATION ACTIONS NOTES/EXAMPLES
• 18
Angina pectoris ~ heart rate and contractility, resulting in ~ 0 2
• 19
consumption
• 20
Myocardial infarction ~ mortality
• 21
I A \ / ................1. . .... ._: ..... - . ...... J......... :a. .. 1..... 1............ IT

6
lock
s
Suspend
0
End Block
Item: 12 of 25 ~ 1 • M k -<:J 1>- Jil ~· !:';-~
QIO: 3885 ..L ar Pre v ious Next Lab~lues Notes Calcula t o r
& &
1
r-\ebi\'olol combines ca rdiac-selecti\'e
2
~ra drenergi c blockade with stimulation of
3 ~3 -receptors (activate nitric oxide synthase in
4 the ,-asculature and ~ SVR)
5
6 FA17 p 309.1

7 Antiarrhythmics- Ietoprolol, propranolol, esmolol, alenolol, timolol, can·edilol.


8 ~blockers (class II)

9 MECHANISM Decrease S and AV nodal acli' il) b) ! cA~ I P, ! Ca 2+ currents. Suppress abnormal pacemakers b}
10 ! slope of phase 4.
11
AV node particularly sensiti\'C- t PR inten·al. Esmolol ,-ery short acting.

12
CLINICAL USE SVT, ''entricular rate control for atrial fi brillation and atrial Rutter.
• 13 ADVERSE EFFECTS Impotence, exacerbation of COPD and asthma, cardiovascular effects (bradycardia, AV block, JIF),
• 14
C 1$ effects (sedation, sleep ~!It erat io ns). May 111ask the signs of hypoglycemia.
Metoprolol can cause dyslipidemia. Propranolol can exacerbate vasospasm in Prinzmetal angina .
• 15
~-bl ocke rs (except the nonsclccti\'c a- and ~ -an t:tgo n is ts can·cdilol and labetalol) cause unopposed
• 16 llr agonism if given alone for pheochromocytoma or cocaine toxicity. Treat ~-bl ocke r overdose with
• 17 saline, atropine, glucagon.
. 18 Class II
Decrease slope Prolonged
• 19 > 60 of phase 4 repolanzation
E. 30 (at AV node)
• 20 ~ depol\n

~
c~
• 21 0
• ..8.
a
Lock
s
Suspend
8
End Bl ock
Item: 12 of 25 ~ 1 • M k -<:J 1>- Jil ~· !:';-~
QIO: 3885 ..L ar Pre v ious Next Lab~lues Notes Calcula t o r
& &
1
FA17 p 273.1
2 Cardiac output variables
3 Stroke volume SLroke Volume affected by Contractility, SV CAP.
4 \fterload, and Preload. A failing heart has l SV (systolic and/or diastolic
5 t SV with: dysfunction)
6
t Contractility (eg, anxiety, exercise)
• t Preload (eg, early pregnancy)
7
• l Afterload
8
Contractility Contractility (and S ) t with: Contractilitr (and SV) l with:
9
• Catecholamine stimulation via ~ 1 receptor: ~ 1 -blockade (l cA~ lP)
10 Ca 2+ channels phosphorylated - t Ca 2.. IIF with systolic dysfunction
11 entry - t Ca 2+-induced Ca 2+ release and cidosis
12 t Ca 2+ storage in sarcoplasmic reticulum Hypoxia/hypercapnia (l Po,lt Pco,)
Phospholamban phosphor)lation - active on-dihydropyridine Ca 2+ ~hanncl blockers
• 13
Ca 2+ ATPase - t Ca 2+ storage in
• 14
sarcoplasmic reticulum
• 15 • t intracellular Ca2+
• 16 • l extracellular a+ U activity of 1 a 1/Ca 2+
• 17 exchanger)
Digitalis (blocks Na+JK+ pump
. 18
..... t intracellular !a+ ..... l Na' /Ca 2+
• 19
exchanger activity ..... t intracellular Ca 2+)
• 20
Myocardial oxygen ~lroC.\RD ial 0 2 demand is t by: Wall tension follows Laplace's law:
• 21 demand
• • t Contractility

a
Lock Suspend
s 8
End Bl ock
Item: 13 of 25 ~ 1 • M k -<:J 1>- Jil ~· !:';-~
QIO: 3212 ..L ar Pre v ious Next Lab~lues Notes Calcula t o r
& &
1
A 58-year-old man with a past medical history of hypertension goes to his physician for a routine visit. On physical examination the physician Is able
2 to detect an 54 heart sound, and refers the patient to a cardiologist. After a thorough work-up, he is found to have left ventricular hypertrophy. The
Image below plots left ventricular pressure versus left ventricular volume for a single cardiac cycle .
3
4
5 f
ISO
6
7 140

8 120
9
10
11
12
c
• 13 B
• 14
20 /
• 15 A
• 16 0'-T----..,.
40 M I~
• 17 V. ntrleul.lr IIOiume !n l
• 18
This patient's 54 heart sound heard on auscultation would best correspond to which of the following points?
• 19
• 20
A
• 21

a
Lock
s
Suspend
8
End Bl ock
Item: 13 of 25 ~ 1 • M k -<:J 1>- Jil ~· !:';-~
QIO: 3212 ..L ar Pre v ious Next Lab~lues Notes Calcula t o r

2 100
3 t 80
4 f•
5
,.,
~

~
60
c
t
6 1:. ~ 8
\
7 20 /
A
8
0
9 <&() IS llO
VOt"ttli<uiM volume lrTt.l
10
11 Th is patient's 54 heart sound heard on auscultation would best correspond to which of the following points?
12
• 13 A

• 14
B
• 15
c
• 16
• 17 0

• 18 E
• 19
F
• 20
• 21

a
Lock
s
Suspend
8
End Bl ock
Item: 13 of 25 ~ 1 • M k -<:J 1>- Jil ~· !:';-~
QIO: 3212 ..L ar Previous Next Lab~lues Notes Calculat or
& &
1 The co rrect a n sw er i s B. 600/o chose this.
2 Point A corresponds to the opening of the mitral valve at the beginning of diastole, and the line from A to c shows the increase in ventricular volume
during diastole. Point C marks the beginning of systole as left ventricular pressure becomes greater than left atrial pressure, causing the mitral valve to
3 close . This closure (in conjunction with the closure of the tricuspid valve) represents 51. The line from point c to D corresponds to isovolumetric
4 contraction, during which both the mitral and aortic valves remain closed as the left ventricular pressure increases. At point D the left ventricular pressure
becomes greater than the aortic pressure and the aortic va lve opens. Between points D and F, the left ventricular pressure continues to increase as the
5 ventricle continues to contract and blood is ejected from the left ventricle Into the aorta. At point F the aortic va lve closes when the left ventricle begins
to relax and the left ventricular pressure becomes less than aortic pressure. This closure (in conjunction with the closure of the pulmonic valve) represents
6 52. The line from po111t F to point A represents the isovolumetric relaxation that occurs after the end of ventricular systole. When the left ventricular
pressure becomes less than the pressure in the left atrium, the mitral valve opens, thus beginning a new loop of the cardiac cycle (diastole plus systole).
7
Point B corresponds to the point near the end of diastole when 54 may be heard. An 54 (which occurs when a stiff ventricle causes turbulent flow In the
8 atria), commonly follows the "atrial kick" (forceful atrial systole, a cause of the 54). It is not normally present in adults. Its presence suggests a decrease
In ventricular compliance, such as occurs in ventricular hypertrophy resulting from chronic hypertension. 54 is thought to result from vibration of a stiff,
9 noncompliant ventricular wall as blood is rapidly ejected into the ventricle.
Pulmonary valve Tricuspid valve Cardiac cycle Systole Diastole Systolic geometry Aorta Aortic valve Hypertension Isovolumic relaxation time ventricle {heart)
10
Atrium {h art) Mitral valve Ventricular hypertrophy Isovolumetric contraction Hypertrophy Heart Tricuspid atresia Ventricular pressure Ventricular system
11
A is not co rrect. 20 0/o chose this.
12
Point A represents the opening of the mitral valve at the beginning of diastole, not an 54 heart sound. Normally no sound is heard when the mitral valve
13 opens. However, in cases of mitral stenosis, an opening click may be audible If the valve leaflets are stiff. In addition, in some cases a third heart sound
(53) may be heard shortly after point A at the beginning of diastole. 53 Is due to the vibration of the distended ventricular wall during rapid filling and ts
• 14 usually soft and low in frequency. Although the presence of an 53 is normal In children, in adults it usually suggests volume overload, such as occurs In
congestive heart failure .
• 15
Third heart sound Heart failure Mitral valve stenosis Congestive heart failure Fourth heart sound Mitral valve Diastole Heart sounds Stenosis volume overload
• 16
C is not co rrect .
• 17 Point C corresponds to 5 1, which is hea rd normally when the mitral and tricuspid valves close at the end of diastole. The 54 sound would be heard just
before this at point B .
. 18 Diastole Tricuspid valve Tricuspid atresia Cusp {anatomy) Olfactory bulb mitral cell
• 19
0 i s n ot correct. 7 % chose this .
• 20 Point D represents the opening of the aortic valve. This normally creates no audible sound on auscultation, but there may be an ejection click at this point
In some cases of aortic stenosis.
• 21 Aortic stenos s Auscultation Aortic valve Stenosis

a
Lock
s
Suspend
8
End Bl ock
Item: 13of25 ~. , . M k <:] t> al ~· ~
QIO: 3212 .l. ar Previous Next lab 'Vfllues Notes Calculator

• Point C corresponds to 5 1, which is heard normally when the mitral and tricuspid valves close at the end of diastole. The 5 4 sound would be heard just •
1
before this at point B.
2 Diastole Tricuspid valve Tricuspid atresia Cusp (anatomy) Olfactory bulb mitral cell

3 D is no t co rrect. 7 % cho se this.

4 Point D represents the opening of the aortic va lve . This normally crea t es no audible sound on auscultation, but there may be an ej ection click at this point
in some cases of aortic stenosis.
5 Aortic stenosis Auscultation Aortic valve Stenosis

6 E is no t co rrect. 13% cho se this.

7 Point E may correspond to an audible ej ection murmur in cases of aortic stenosis. However; this is not the point in the cardiac cycle when one expects to
hear an 5 4 heart sound.
8 Cardiac cycle Aortic stenosis Fourth heart sound Stenosis Heart sounds Heart murmur

9 F is no t co rrect.

10 Point F represents the sound of the aortic va lve closing when the left ventricle begins to relax and the left ventricular pressure becomes less than aortic
pressure. The closure of the aortic valve (in conjunction with the closure of the pulmonic va lve) can be heard on auscultation as the second heart sound
11 (52).
Pulmonary valve Auscultation Ventricle (heart) Aortic valve Heart sounds Ventricular pressure Heart left ventricular pressure
12
13
• 14 Botto m Li ne:
An 5 4 heart sound, commonly called the " atrial kick," is not normally present in adults; its presence suggests a decrea se in ventricular compliance such
• 15
occurs as in ventricular hypertrophy resulting from chronic hypertension. 5 4 is thought to result from vibration of a stiff, noncompliant ventricular wall as
• 16 blood is rapidly ej ected into the ventricle from the atrium .
Fourth heart sound Hypertension Heart sounds Ventricle (heart) Ventricular hypertrophy Atrium (heart) Hypertrophy
• 17

• 18
• 19
lijl;fiiJI•l toryear:[2017 • ]
FI RST AID FAC T S
• 20
• 21
• .- .... .., - , ..,0 •

6
lock Suspend
s 0
End Block
Item: 13of25 ~. , . M k <:] t> al ~· ~
QIO: 3212 .l. ar Previous Next Lab 'Vfll ues Notes Calculator

2
3 lijl;fiiJI•l toryear:[2017 • ]
FI RST AID FAC T S
4
5
FA17 p276.1
6
Pressure-volume loops and cardiac cycle The black loop represents normal cardiac
7 physiology.
140
8 I Contractility I Afterload
9 t sv I (Aortic pressure)
120 tEF I SV Phases- left ventricle:
10 0.
:I:
IESV I ESV 0 lsovolumetric contraction-period
~ 100 between mitral valve closing and aortic
11
valve opening; period of highest 0 2
~
12
13
1 so
~ Stroke
opens
consumption
E} Systolic ejection-period between aortic
.!!!
volume Pr lo d valve opening and closing
• 14 .§ 60 (EDv-ESV) I SV E) Isovolumetric relaxation-period between
• 15 i Mitral
aortic valve closing and mitral valve
• 16
§ 40 valve
open tng
/ opens ()& 0 Rapid fil ling-period just after mitral
• 17 20 0 valve opening
• 18 S3 S1 0 Reduced fi ll ing-period just before mitral
valve closing
• 19 ESV Left ventricular volume EDV

• 20
• 21 Systole Diastole

6
lock
s
Suspend
0
End Block
Item: 13 of 25 ~ 1 • M k -<:J 1>- Jil ~· !:';-~
QIO: 3212 ..L ar Pre v ious Next Lab~lues Notes Calcula t o r

3 FA17 p278.1

4 Auscultation of the heart


5
6 Where to bsten. APT M

7 Aortic area:
8 Systolic murmur Pulmonic area:
Aort stenos1s Systolic ejection murmur
9 Flow murmur PulmOOIC stenosiS
(!!(]. phywlogiC murmur) Row murmur
10
Aortoe valve sc eros1s
Tricuspid area:
11
border: --~----------~~-­
Holosystolic murmur
12 left sternal
DiastoliC murmur Tncuspld ri!(Jurg1tation
AortiC ri!(Jurg1tat1on Ventncular septal defect
13 Diastolic murmur
Pulmon c ri!(Jurg1tat'on
• 14 SystoliC murmur Tncuspld stenos1s
HypertrophiC Atnal septal defect (i llow
• 15 cardiomyopathy @- across tncusp1d valve)
• 16 - - - - - - - - - Mitral area (apex):
- AOftic
• 17 - Putmomc Holosystolic murmur
- Trkusp~d M1tral ri!(Jurgitation
. 18 Systolic murmur
- Mnral
Mrtral valve prolapse
• 19
Diastolic murmur
• 20 M1tral stenosis
• 21
• fHffT

a
Lock
s
Suspend
8
End Bl ock
Item: 14 of 25 ~ 1 • M k -<:J 1>- Jil ~· !:';-~
QIO: 10 37 ..L ar Pre v ious Next Lab~lues Notes Calcula t o r
& &
1
You are caring for a 35-year-old male patient in the ICU for septic shock. Norepinephrine is being administered to address his hemodynamic status.
2 The resident explains to the student on the t ea m that the sympathetic effects on the heart are enacted via the neurotransmitter norepinephrine
acting on 13 1 -receptors. It has positive chronotropic, dromotropic, and Inotropic effects.
3
4 Which of the following sympathetic effects is correctly paired with its physiologic mechanism?
5
6 A. Pos1t1ve chronotropic effect: decreased inward Na+ current in the SA node
7
B. Positive chronotropic effect: increased outward Na+ current In the SA node
8
c. Positive dromotropic effect: increased inward ca 2 + current in the AV node
9
10 D. Positive dromotropic effect : increased inward Na+ current in the AV node

11 E. Positive inotropic effect : decreased inward ca 2 + current in myocytes


12
13
• 14
• 15
• 16
• 17
• 18
• 19
• 20
• 21

a
Lock
s
Suspend
8
End Bl ock
Item: 14 of 25 ~. I • M k <:] t> al ~· ~
QIO: 1037 .l. ar Previous Next Lab 'Vfll ues Notes Calculator

1
The co rrect a nswer is c . 48% cho se this.
2
A positive dromotropic effect is an increa se in conduction velocity through the AV node and His-Purkinje system . Norepinephrine acts on j3 1 -adrenergic
3 receptors and, through G-protein activation, lea ds to increa se c a 2 + influx into the cells. This decrea ses the potential needed to cause depolarization of
these cells.
4 Dromotropic Norepinephrine Atrioventricular node Depolarization Electrical conduction system of the heart Adrenergic receptor Receptor (biochemistry)
5 A is no t co rrect. 9 % cho se this.
6 Positive chronotropic effect, which is an increa se in heart rat e, is caused by an increa se in the firing rat e of the SA node due to an increa se in the inward
Na + current during phase 4 of the nodal action potential.
7 Action potential Chronotropic Heart rate Sinoatrial node
8 B is no t co rrect. 1 6 % cho se this.
9 Positive chronotropic effect is caused by an increa se in the firing rat e of the SA node due to an increa se in the inward Na + current, not outward
Na + current, which is responsible for repolarization along with potassium efflux.
10 Chronotropic Sinoatrial node Potassium
11 D is no t co rrect. 23 % cho se this.
12 Positive dromotropic effect is an increa se in conduction velocity through the atrioventricular node via voltage gat ed Ca 2 + channels, is caused by an
increa se in the inward c a 2 + current. Inward Na + current is responsible for increa sing the firing rate of the sinoatrial node and an increa se in heart rat e.
13 Atrioventricular node Sinoatrial node Dromotropic Heart rate Nerve conduction velocity Voltage
14
E is no t co rrect. 4 % cho se this.
• 15 Positive inotropic effect, which is an increa se in contractility, is caused by an increa se in the inward c a 2 + current in myocytes
Inotrope Contractility Myocyte
• 16
• 17

• 18 Botto m Li ne:
Inotropes increa se contractility, chronotropes increa se sinoatrial node firing, and dromotropes increa se conduction velocity through the atrioventricular
• 19
node.
• 20 Atrioventricular node Sinoatrial node Contractility Inotrope Nerve conduction velocity

• 21

6
lock
s
Suspend
0
End Block
Item: 14 of 25 ~. I • M k <:] t> al ~· ~
QIO: 1037 .l. ar Previous Next Lab 'Vfll ues Notes Calculator

1
Bottom Line:
2
Inotropes increase contractility, chronotropes increase sinoatrial node firing, and dromotropes increase conduction velocity through the atrioventricular
3 node.
Atrioventricular node Sinoatrial node Contractility Inotrope Nerve conduction velocity
4
5
6
I iii I;fi 1!1 I•J for year:l 2017 ..
7 FI RST AI D FA CT S

8
9 FA17 p273.1
Cardiac output variables
10
Stroke volume Stroke Volume affected by Contractility, SVCAP.
11
Afterload, and Preload. A failing heart has l SV (systolic and/or diastolic
12
t SV with: dysfun ction)
13 • t Contractility (eg, anxiety, exercise)
14 • t Preload (eg, early pregnancy)
• 15 • l Afterload
• 16 Contractility Contractility (and SV) t with: Contractility (and SV) l with:
• 17 • Catecholamine stimulation via ~ 1 receptor: • ~ 1 -blockad e (l cAMP)
Ca 2+ channels phosphorylated -+ t Ca 2+ • HF with systolic dysfunction
• 18
entry -+ t Ca 2+-induced Ca 2+ release and • Acidosis
• 19
t Ca 2+ storage in sarcoplasmic reticulum • Hypoxia/hypercapnia (l Poz!t Pco2)
• 20 Phospholamban phosphorylation -+ active • :--Jon-dihydropyridine Ca 2+ chan nel blockers
• 21 Ca 2+ ATPase -+ t Ca 2+ storage in

6
lock Suspend
s 0
End Block
Item: 14 of 25 ~ 1 • M k -<:J 1>- Jil ~· !:';-~
QIO: 10 37 ..L ar Pre v ious Next Lab~lues Notes Calcula t o r

1 contractility. •

3 FA17 p234.1
4 Sympathomimetics
DRUG ACTION APPLICATIONS
5
6 Direct sympathomimetics
7 Albuterol, salmeterol l3z > P1 Albuterol for acute asthma or COPD. Salmeterol
for long-term asthma or COPD control.
8
9 Dobutamine llearl failure (JJF) (inotropic> chronotropic),
cardiac stress testing.
10
Dopamine Unstable bradycardia, HF, shock; inotropic and
11
chronotropic effects at lm\·er doses due to p
12
effects; vasoconstriction at high doses due to a
13 effects.
14 Epinephrine P> a Anaphylaxis, asthma, open-angle glaucoma;
• 15 a effects predomi nate at high doses.
• 16 Significantly stronger effect at Pz-receptor than
• 17 norepinephrine.

. 18 Fenoldopam Postoperative hypertension, hypertensi\'e crisis.


Vasodilator (coronary, periphera l, renal, and
• 19
splanchnic). Promotes natriuresis. Can cause
• 20
hypotension and tachycardia.
• 21
• lsooroterenol F. Ieel ronll\·sioloPic evaluation of

a
Lock
s
Suspend
8
End Bl ock
Item: 14 of 25 ~ 1 • M k -<:J 1>- Jil ~· !:';-~
QIO: 10 37 ..L ar Pre v ious Next Lab~lues Notes Calcula t o r

1 YP )

2 Ephedrine Indirect general agonist, releases stored asal decongestion (pseudoephedrine), urinar)
catecholamines incontinence, hrpotension.
3
4
5 FA17 p 281 .1

6 Pacemaker action Occurs in the SA and AV nodes. Key differences from the \entricular action potential include:
7 potential
Phase 0 =upstroke-opening of ,-oltagc-gated Ca 2+ channels. Fast voltage-gated a+ channels are
8 permanently inactivated because of I he less negative resting potential of these cells. Results in a slm\
9 conduction velocity that is used by the A node to prolong transmission from the atria to \'Cntricles.
10 Phases I and 2 arc absent.
11
Phase 3 =inactivation of the Ca 2+ channels and f activation of K+ channels - f K+ cfAux.
12
Phase 4 = slow spontaneous diastolic depolarization due to lr ("funny current"). lr channels
13
responsible for a slow, mixed I a+fK+ inward current; different from 1~3 in phase 0 of ventricular
14 action potential. Accounts for automaticity of SA and A nodes. The slope of phase 4 in the SA
• 15 node determ ines HR. ACh/adenosine l the rate of diastolic depolarization and l HR, while
• 16 catecholamines f depolarization and f IIR. Sympathetic stimulation f the chance that lr channels
are open and thus f I IR .
• 17
• 18
0 ------------------------- -- ------------·
• 19
-20
• 20 ~
=" - 40
• 21 :i

a
Lock Suspend
s 8
End Bl ock
Item: 15 of 25 ~ 1 • M k -<:J 1>- Jil ~· !:';-~
QIO: 4552 ..L ar Pre v ious Next Lab~lues Notes Calcula t o r
& &
1
Cardiac muscle cells have novel m echanisms of action potential generation and contraction compared with skelet al or smooth muscle cells.
2
3 Catecholamine's effect on Phospholamban directly regulates which of the following in cardiac myocytes?
4
5 A. Caldum binding affinity of troponin C

6 B. CalciUm 1nflux into the terminal dsterna and sarcoplasmic reticulum


7
c. Dephosphorylation of the myosin cross bridge
8
D. Gap junction permeability between myocytes
9
10 E. Myosin binding to actin molecule

11 F. Sodium ion influx through slow sodium channels


12
13
14
0 15
0
16
0
17
0
18
• 19
0 20
• 21

a
Lock
s
Suspend
8
End Bl ock
Item: 15of25 ~. , . M k <:] t> al ~· ~
QIO: 4552 .l. ar Previous Next lab 'Vfl1 ues Notes Calculator

1
The co rrect a nswer is B. 56 % cho se this.
2
L-type calcium channels relea se a small amount of calcium when an action potential is generat ed. This calcium induces opening of rya nodine receptors
3 (calcium-induced calcium channels) and relea se of calcium from the t erminal cisternae (specialized portions of the sarcoplasmic reticulum) into the
cytoplasm. After myosin cross-bridge formation and ATP hydrolysis, the sarcoplasmic reticulum calcium ATPase (SERCA2) pumps calcium back into the
4 sarcoplasmic reticulum from the cytoplasm. This calcium transport is inhibited when phospholamban is dephosphorylat ed. Dephosphorylat ed
5 phospholamban binds to SERCA2 and decrea ses its affinity for calcium. When phospholamban is phosphorylat ed by PKA inhibition, however; the
sarcoplasmic reticulum calcium pump (SERCA) is lost. Activators of PKA, such as the j3 -adrenergic agonist epinephrine (relea sed by sympathetic
6 stimulation), may enhance the rat e of cardiac myocyte relaxation . In addition, since SERCA is more active, the next action potential will cause an
increa sed relea se of calcium, resulting in increa sed contraction (positive inotropic effect).
7 Action potential Myocyte Epinephrine SERCA Phospholamban Cytoplasm Myosin Sarcoplasmic reticulum Calcium Calcium ATPase Protein kinase A ATPase
8 l-type calcium channel Calcium channel Adenosine triphosphate Inotrope Phosphorylation Voltage-dependent calcium channel Cardiac muscle cell Hydrolysis

9 Agonist ATP hydrolysis Cavl.l Sympathetic nervous system ATP2A2 Ryanodine

10 A is no t co rrect. 10 % cho se this.


Phospholamban does not affect the affinity of troponin C for calcium. This affinity can be altered by troponin activators (calcium sensitizers) or inhibitors
11 (calcium desensitizers) .
12 Phospholamban Troponin c type 1 Troponin c Calcium Troponin

13 c is no t co rrect. 10 % cho se this.


Phospholamban is a phosphoprotein that binds SERCA2 and is not involved in myosin cross-bridge formation .
14
Phospholamban Myosin ATP2A2 SERCA
15
D is no t co rrect. 8% cho se this •
• 16 Phospholamban does not regulat e gap junction permeability. These gap junctions in cardiac myocytes can be opened using rotigaptide.
Gap junction Phospholamban Rotigaptide Cardiac muscle Cardiac muscle cell Myocyte
• 17

• 18 E is no t co rrect. 6 % cho se this •


Phospholamban does not directly regulat e the binding of myosin to actin . The binding of ATP to myosin can reduce myosin's affinity for actin .
• 19 Phospholamban Actin Myosin Adenosine triphosphate
• 20 F is no t co rrect. 10 % cho se this.
• 21 Phospholamban does not bind to or regulat e sodium channels in cardiac muscle .

6
lock
s
Suspend
0
End Block
Item: 15of25 ~. , . M k <:] t> al ~· ~
QIO: 4552 .l. ar Previous Next lab 'Vfl1 ues Notes Calculator
.- J
1
E is not correct. 6% chose this .
2 Phospholamban does not directly regulate the binding of myosin to actin . The binding of ATP to myosin can reduce myosin's affinity for actin .
3 Phospholamban Actin Myosin Adenosine triphosphate

4 F is not correct. 10% chose this .


Phospholamban does not bind to or regulate sodium channels in cardiac muscle.
5
Phospholamban Cardiac muscle Sodium Muscle
6
7
Bottom Line:
8
Dephosphorylated phospholamban binds to sarcoplasmic reticulum calcium ATPase, which pumps calcium back into the sarcoplasmic reticulum from the
9 cytoplasm and decreases its affinity for calcium.
Phospholamban Calcium ATPase Cytoplasm Sarcoplasmic reticulum ATPase Calcium Dephosphorylation Phosphorylation
10
11

12
lijj ;fi IJ l•l for year:l 2017 ..
13 FI RST AI D FA CTS

14
FA17 p433.1
15
• 16 Muscle conduction to T-tubules are extensions of plasma membrane juxtaposed with term inal cisternae of the
• 17
contraction sarcoplasmic reticulum, allowing for coordinated contraction of muscles.
Dihydropyridine receptor In skeletal muscle, 1 T-tubule + 2 term ina I cisternae = triad.
• 18
1-.!UIJUle membrane In cardiac muscle, I T·tubulc + I terminal cisterna= dyad.
• 19 I. Action potential depolarization opens presynaptic voltage-gated Ca 2+ channels, inducing
• 20 neurotransmitter release.
• 21 2. Postsynaptic ligand binding leads to muscle cell depolarization in the motor end plate .
, I ,...., , 1 1
" ' '
6
lock
s
Suspend
0
End Block
2
Aband II I band I
3
u
4 Hband

5
6 FA17 p 280.1
7 Myocardial action Also occurs in bundle of His and Purkinje fibers.
8 potential
Phase 0 =rapid upstroke and dcpolarization-,·oltage-gated ~a+ channels open.
9
Phase 1 = initial repolarization-inactivation of ,·oltage-gated i\a+ channels. Voltage-gated K+
10
channels begin to open.
11
Phase 2 = plateau-Ca 2+ inAux through ,·oltage-gated Ca2+ channels balances K+ efAux. Ca2+
12
influx triggers Ca2+ release from sarcoplas111ic reticulum and myocyte contraction.
13
Phase 3 =rapid repolarization - massive K+ efnu x due to opening of voltage-gated slow K+
14
channels and closure of voltage-gated Ca21 channels.
15
Phase 4 = resting potential- high K'" permeability t·hrough K+ channels.
• 16
• 17
In contrast to skeletal muscle:
• 18 Cardiac muscle action potential has a plateau, wh ich is due to Ca 2+ inAux and K+ efRux.
• 19 Cardiac muscle contraction requires Ca 2+ inOux from ECF to induce Ca2+ release from
• 20 sarcoplasm ic reticulum (Ca2.,.-induced Ca 2.,. release).
Cardiac myocytes are electrically coupled to each other by gap junctions.
• 21

a
Lock
s
Suspend
8
End Block
Item: 15 of 25 ~ 1 • M k -<:J 1>- Jil ~· !:';-~
QIO: 4552 ..L ar Pre v ious Next Lab~lues Notes Calcula t o r

1
& K Na+ Ca &

......____ I /
2 "Leak" currents
3
4 FA17 p 273.1
5 Cardiac output variables
6 Stroke volume SLroke \'olume affected b) Contractilit), SVCAP.
7 -\fterload, and Preload. A fa iling heart has l SV (systolic and/or diastolic
t sv \\·ith: dysfunction)
8
t Contractility (eg, anxiety, exercise)
9
• t Preload (eg, early pregnancy)
10 • l Afterload
11
Contractility Contractility (and SV) t with: Contractility (and SV) l with:
12 • Catecholamine stimulation via ~I receptor: ~ 1 -blockade (l cA~ JP)
13 Ca2+ channels phosphorylated - f Ca 2+ !IF with systolic dysfunction
14 entry - f Ca 2+-induced Ca 2+ release and • cidosis
f Ca 2+ storage in sarcoplasmic reticulum • Hypoxia/hypercapnia (l Po,/f Pco,)
15
Phospholamban phosphorylation - active • Non-dihydropyridine Ca 2+ ~hannel blockers
• 16
Ca2+ ATPase - t Ca 2+ storage in
• 17 sarcoplasmic reticulum
. 18 t intracellular Ca2+
• 19 • l extracellular f a+ (l activity of a' /Ca 2•
• 20
exchanger)
Digitalis (blocks 1 a+f K+ pump
• 21
• - f intr::~cellnl:u '::~+ - l ::~+ /C:::~2+

a
Lock Suspend
s 8
End Bl ock
Item: 16 of 25 ~ 1 • M k -<:J 1>- Jil ~· !:';-~
QIO: 4875 ..L ar Pre v ious Next Lab~lues Notes Calcula t o r
& &
1
A 75-year-old nursing home resident is brought to the emergency department because of confusion and reduced m ental alertness. caregivers say
2 that the patient had developed a productive cough with green sputum a few days ago and now has shaking chills and fever. Physical examination
reveals an Ill-appearing gentleman with a temperature of 38.9°C ( 102°F), heart rate of 110/min, blood pressure of 90/60 mm Hg, and oxygen
3 saturation of 92% on room air. He appears flushed, and his pulses are bounding. Laboratory values are significant for a WBC count of 14,000/mm•, a platelet
count of 50,000/mm•, and elevated serum lactic acid levels. (Reference range: Normal CVP is 2-8 mm Hg, normal PCWP is 4-1 2 mm Hg, normal co Is 4-8
4 Ljmln, and normal SVR is 700-1 600 dyn/ sec/cm-5 .)
5 Central Pulmonary Systemic
Venous Capillary Cardiac Vascular
6
Choice Pressure Wedge Output Resistance
7 (mm Hg) Pressure (Umin) (dyne/sec/em~ )
(mm Hal
8
A Normal Normal .1. t
9
B Normal Normal
t .1.
10
11 c Normal
t t .1.
12 t .1. t
D Normal
- t
13
E t t .1.
14
15 Which physiologic parameters from the t able above would the physician expect to find In this patient?
• 16
• 17 A

. 18 B
• 19
c
• 20
D
• 21

a
Lock
s
Suspend
8
End Bl ock
Item: 16 of 25 ~ 1 • M k -<:J 1>- Jil ~· !:';-~
QIO: 4875 ..L ar Pre v ious Next Lab~lues Notes Calcula t o r

•1 &
count of 50,000/mm•, and elevated serum lactic acid levels. (Reference range: Normal CVP is 2-8 mm Hg, normal PCWP is 4-1 2 mm Hg, normal co Is 4-8 &

L/mln, and normal SVR is 700-1 600 dyn/sec/cm-s)


•2
Central Pulmonary Systemic
•3 Venous Capillary Cardiac Vascular
Choice Pressure Wedge Output Resistance
·4 (mm Hg) Pressure (Umin) (dyne/sec/em-S )
•5 Cmm Hal

•6 A Normal Normal J. t
•7 8 Normal Normal
t !
·8
c t t J.
Normal .
.9 t J. t
• 10
0 Normal ,. t-

E t t J. t
• 11
• 12
Which physiologic parameters from the table above would the physician expect to find in this patient?
• 13
• 14 A
• 15
B
• 16
• 17 c
• 18 0
• 19
E
• 20
• 21

a
Lock
s
Suspend
8
End Bl ock
Item: 16 of 25 ~ 1 • M k -<:J 1>- Jil ~· !:';-~
QIO: 4875 ..L ar Prev ious Next Lab~lues Notes Calculat o r

& &
1 Th e co rrect an sw er i s B. 460/o ch ose this.
2 This patient is suffering from septic shock secondary to an infection which probably originated in the pulmonary system . Elevated wac, productive cough,
fever and shaking chills, along with the hypotension and tachycardia together suggest septic shock. Septic shock is characterized by high cardiac output
3 (CO) and low systemic vascular resistance (SVR). In the initial stages of sepsis, the heart is able to continue to m aintain co by increasing inotropy. One
4 would expect a low SVR because inflammatory mediators released will lead to vasodilation, as evidenced by this patient's flushed appearance. The central
venous pressure (CVP) and the pulmonary capillary wedge pressure (PCWP) generally correlate with each other and may be low, normal, or high In septic
5 shock. Normal CVP is 2-8 mm Hg, normal PCWP is 4 -1 2 mm Hg, normal CO Is 4-8 I./min, and normal SVR is 700- 1600 dyn/ sec/cm· 5 .
Cent a ve1 ous pressure Septic shock Sepsis Vasodilation cardiac output Hypotension l:lc. ycardia Vascular resistance Pulmonary wedge pressure capi ary
6
lnotrope I ammation Respiratory system Cough Infection Fever Vein Chills Blood vessel
7
A i s not correct. 110/o chose this.
8 These findings would occur in patients with hypovolemic shock. These patients have depletion of their intravascular compartment, leading to low cardiac
9 output (CO). The systemic vascular resistance is high in an effort to maintain the perfusion blood pressure despite decreased co.
Hypovolemia Cardiac output Vascular resistance Blood pressure Perfusion Blood vessel
10
C i s n ot co rrect. 190/o chose this.
11 Pulmonary capillary wedge pressure (PCWP) and central venous pressure (CVP) generally correlate with each other (except in some highly specialized
12 cases beyond the scope of the USMLE Step 1). Therefore, the scenario of a normal CVP (8 mm Hg) in the setting of a high PCWP (14 mm Hg) such as In
choice C would be unlikely.
13 Central venous pressure Christian Democratic People's Party of Switzerland

14 D is not co rrect. 100/o chose this.

15 Pulmonary capillary wedge pressure (PCWP) and central venous pressure (CVP) generally correlate with each other (except in some highly specialized
cases beyond the scope of the USMLE Step 1). Therefore, the scenario of a high CVP ( 12 mm Hg) and a normal PCWP (8 mm Hg) would be unlikely.
16 Furthermore, septic shock is characterized by high cardiac output and low systemic vascular resistance, which is not the case here .
Central venous pressure Septic shock Cardiac output Pulmonary wedge pressure Vascular resistance Capillary Christian Dem ocratic People's Party of Switzerland
• 17
E is n o t co rrect. 140/o ch ose this .
. 18
These findings would occur in patients with cardiogenic shock. Cardiac output Is Intrinsically low in this condition, and systemic vascular resistance will
• 19 Increase due to sympathetically mediated vasoconstriction, leading to a cold and clammy patient. The decreased forward flow of blood due to decreased
function of the heart's ability to pump leads to increased central venous pressure and pulmonary capillary wedge pressure .
• 20 Cardiogenic shoe~ Central venous pressure Vasoconstriction cardiac output Pulmonary wedge pressure Vascular resistance capillary Vein Heart Blood vessel
• 21

a
Lock
s
Suspend
8
End Block
Item: 16of25 ~. , . M k <:] t> al ~· ~
QIO: 4875 .l. ar Previous Next lab 'Vfl1ues Notes Calculator

1
E is not correct. 14% chose this.
2 These findings would occur in patients with cardiogenic shock. Ca rdiac output is intrinsically low in this condition, and systemic vascular resistance will
increase due to sympathetically mediated vasoconstriction, leading to a cold and clammy patient. The decreased forward flow of blood due to decreased
3 function of the heart's ability to pump leads to increased central venous pressure and pulmonary capillary wedge pressure.
4 Cardiogenic shock Central venous pressure Vasoconstriction Cardiac output Pulmonary wedge pressure Vascular resistance Capillary Vein Heart Blood vessel

5
6 Bottom Line:
7 High cardiac output and low systemic vascular resistance (SVR) are indicative of septic shock. The SVR is low because the release of inflammatory
mediators causes vasodilation .
8 Septic shock Cardiac output Vasodilation Vascular resistance Inflammation

9
10
11 I ill ;fi 1!1 I•J for year:[ 2017 ..
FI RST AID FA CTS
12
13 FA17p299.1
14
Shock Inadequate organ perfusion and delivery of nutrients necessary for normal tissue and cellular
15 funct ion. Initially may be reversible but life threatening if not treated promptly.
16
PCWP SVR
• 17 CAUSED BY SKIN (PRELOAD) co (AFTERLOAO) TREATMENT
• 18 Hypovolemic Hemorrhage, dehydration, Cold, u ! t lV fluids
• 19 burns clammy
• 20 Cardiogenic Acute Ml, HF, valvular lnotropes, diuresis
• 21 dysfun ction, arrhythmia
• -·--- -~
,.., 1. Cold, • '' • n ,. 1 . •

6
lock
s
Suspend
0
End Block
Item: 16 of 25 ~ 1 • M k -<:J 1>- Jil ~· !:';-~
QIO: 4875 ..L ar Pre v ious Next Lab~lues Notes Calcula t o r

1 pulmonary embolism,
2 tension pneumothorax
3 Distributive Sepsis, anaphylaxis Warm l t IV Au ids, pressors
4 C 'S injury Dr) l l

5
6 FA17 p 272.1
7 Cardiac output CO= stroke volume (SV) x heart rate (IIR) During the early stages of exercise, CO is
8
rick principle: maintained by t HR and t SV During the late
9 stages of exercise, CO is maintained by t HR
CO = rate of 0 2 consumption
10 only (SV plateaus).
arterial 0 2 content- \'enons 0 2 content
Diastole is preferentially shortened with t IIR;
11
\!lean arterial pressure ( \1 P) =COx total less filling time - l CO (eg, ventricu lar
12
periphera I resist a nee (TPR) tachycardia),
13
\llAP = 2;{ diastolic pressure+ Xsystolic pressure
14
15 Pulse pressure = systolic pressure - diastolic pressure t pulse pressure in hyperthyroidism, aortic
Pulse pressure is proportional to S , inversely regurgitation, aortic stiffening (isolated systolic
16
proportional to arterial compliance, hypertension in elderly), obstructive sleep
• 17
apnea (t sympathetic tone), exercise (transient),
S =end-diastolic ,-olume (EDV) - end-systolic
• 18 l pulse pressure in aortic stenosis, cardiogenic
,-olume (ESV)
• 19 shock, cardiac tamponade, advanced heart
• 20 failure (H F'),
• 21

a
Lock
s
Suspend
8
End Bl ock
Item: 17 of 25 ~ 1 • M k -<:J 1>- Jil ~· !:';-~
QIO: 10 31 ..L ar Pre v ious Next Lab~lues Notes Calcula t o r

1 &

A 55-year-old man with hypertension is prescribed an antiarrhythmic agent that alters the flow of cations in myocardial tissue . The image is a trace of
a myocardial action potential. Each phase is associated with the opening and/or closing of various ion channels.
IA•A] &

3 Ventricular muscle fiber


4
2
5
6
30 ms
1----l
7
0
8
9
10
4
11

12
13 Which of the following is affected by the patient's antiarrhymic agent acting on phase 0 of the myocardial action potential?

14
A. Ligand-gated calcium channels opening
15
16 8. Ligand-gated potassium channels closing

• 17 c. Ligand-gated sodium channels opening


. 18
o. Voltage-gated calcium channels opening
• 19
E• Voltage-gated potassium channels closing
• 20
• 21 F. Voltage-gated sodium channels closing

a
Lock
s
Suspend
8
End Bl ock
Item: 17 of 25 ~ 1 • M k -<:J 1>- Jil ~· !:';-~
QIO: 10 31 ..L ar Pre v ious Next Lab~lues Notes Calcula t o r
& &
1
2
2
3
30ms
~
4
0
5
6
7
4
8
9
10 Wh ich of the followi ng is affected by the patient's antiarrhymic agent acti ng on phase 0 of the myocardial action potential?

11
A. Ligand-gated calcium channels opening
12
13 B. Ligand-gated potassium channels closing

14 c. Ligand-gated sodium channels opening


15
D. Voltage-gated calcium channels opening
16
• 17 E. Voltage-gated potassium channels closing

• 18 F. Voltage-gated sodium channels closing


• 19
G. Voltage-gated sodium channels opening
• 20
• 21

a
Lock
s
Suspend
8
End Bl ock
Item: 17 of 25 ~ 1 • M k -<:J 1>- Jil ~· !:';-~
QIO: 10 31 ..L ar Prev ious Next Lab~lues Notes Calculat o r

& &
1 Th e co rrect an sw er i s G. 6 1% chose this.
2 Voltage-gated sodium channels open initially at phase o. Voltage-gated ca 2 + channels (L type) open slowly
In response to the sodium upstroke (approximately around - 40 mV), allowing ca 2 + to flow down its
3 concentration gradient and into the cell. Concurrently, there is an outward K+ current via voltage-gated
4 channels that leads to the plateau. The result is a slow conduction velocity that prolongs the transmission
from the atria to the ventricles. Different class 1 drugs can affect the absolute refractory period (ARP). For
5 class lA, It will prolong the ARP as it increases the action potential duration. For class lB it will decrease __...
II' ........_,..
the action potential and therefore decrease the ARP. Lastly for class lC, there Is no change 111 the ARP.
6 I
Regard less of ARP action, all class 1 drugs decrease the slope of phase 0. I
Action poteo to a Atroum heart) Sodium Sodium channel Refractory period physiology1 veotncle ,heart) I
7 JIC
L-type calcium channel Molecular diffusion Voltage-gated ion channel I
8 I
I
9 I

10

11
Ventricular action potenbal

12
A is not co rrect. 30/o chose t his.
13
Ion channels In the myocardium are voltage gat ed, not ligand gated like nicotinic acetycholine receptors.
14 Cardiac muscle Ugand (biochemistry) Ugand Ion channel Ion Voltage Receptor (biochemistry)

15 B is not co rrect . 20/o chose this.


During phase 2, there is an inactivation of voltage -gat ed sodium channels, and opening of voltage-gated potassium channels to allow potassium efflux.
16
Potassium Sodium Potassium channel Sodium channel Voltage-gated potassium channel Efflux (microbiology) Voltage- gated ion channel
17
C i s n o t co rrect . 1 1 0/o ch ose this .
• 18 Ion channels in the myocardium are voltage gated, not ligand gated like nicotinic acetycholine receptors.
Cardiac muscle Ugand (biochemistry) Ugand Ion channel Ion Voltage Receptor (biochemistry)
• 19

• 20 D i s n ot correct. SOfa ch ose this.


Voltage-gated sodium channels are responsible for the upstroke (depolarization) in ventricular cells (phase O) .
• 21 Depolarization Ventncular syncytium Sodium Sodium channel Voltage-gated ion channe Voltage-gated sodium channels

a
Lock
s
Suspend
8
End Block
Item: 17 of 25 ~. I • M k <:] t> al ~· ~
QIO: 1031 .l. ar Previous Next Lab 'Vfll ues Notes Calculator

1 c is no t co rrect. 11% cho se this.


2 Ion channels in the myocardium are voltage gat ed, not ligand gat ed like nicotinic acetycholine receptors.
Cardiac muscle ligand (biochemistry) ligand Ion channel Ion Voltage Receptor (biochemistry)
3
D is no t co rrect. 5 % cho se this.
4
Voltage -gat ed sodium channels are responsible for the upstroke (depolarization) in ventricular cells (phase 0 ).
5 Depolarization Ventricular syncytium Sodium Sodium channel Voltage-gated ion channel Voltage-gated sodium channels

6 E is no t co rrect. 2 % cho se this.


During phase 2, voltage -gat ed potassium cells open to allow K+ efflux. The plat eau occurs because c a 2 + influx balances the K+ efflux.
7
Potassium Efflux (microbiology) Voltage-gated ion channel
8
F is no t co rrect. 16% cho se this.
9 Voltage-gat ed sodium channels are responsible for the upstroke in ventricular cells (phase 0 ). These open in response to depolarization to the - 55 -mV
threshold value, allowing Na+ to rapidly flow down its concentration gradient into the cell. These channels are then inactivat ed and cannot be opened
10
again until the cell is repolarized.
11 Depolarization Sodium Molecular diffusion Sodium channel Ventricular syncytium Voltage-gated ion channel Ventricle (heart) Gradient Ventricular system

12
13 Bo tto m Line:
14 During the myocardial action potential, keep in mind which channels are open and closed during ea ch phase. During phase 0 , voltage-gat ed sodium
channels open, lea ding to a massive Na+ influx that rapidly depolarizes the cell.
15 Action potential Sodium Sodium channel Voltage-gated sodium channels Depolarization

16
17
• 18 lijj ;fi IJ l•l f o r year:l 2 0 1 7 ..
FI RST AI D FAC TS
• 19
• 20
FA17p280.1
• 21
Mvocardial action A lso o rrnrs in hn nrllf' o f H is :mrl Pnrkin if' fiht> r.<.

6
lock
s
Suspend
0
End Block
Item: 18 of 25 ~ 1 • M k -<:J 1>- Jil ~· !:';-~
QIO: 10 38 ..L ar Pre v ious Next Lab~lues Notes Calcula t o r

1 &

An 18-year-old male with a known cardiac arrhythmia presents to the outpatient clinic and states that his heart occasionaly "beats very quickly." He
states that these episodes occur suddenly and that he can alleviate the problem by massaging his neck or splashing cold water on his face. He Is told
IA•A] &

2
that In some patients with arrhythmias, temporary management of the condition can be achieved via carotid sinus massage.
3
4 Which of the following effects is a direct result of this maneuver on the parasympathetic nervous system?
5
6 A. Decreased PR interval
7
B. Increased conduction velodty through the sinoatrial node
8
c. Increased peripheral vascular resistance
9
10 D. Increased ventricular contractility

11 E. Slowed sinoatrial node automaticity


12
13
14
15
16
17
. 18
. 19
• 20
. 21

a
Lock
s
Suspend
8
End Bl ock
Item: 18of25 ~. , . M k <:] t> al ~· ~
QIO: 1038 .l. ar Previous Next lab 'Vfl1ues Notes Calculator

1
The co rrect a nswer is E. 79% cho se this.
2
The baroreceptors in the carotid sinus respond to stretching of the arterial wall. If the arterial pressure suddenly rises, the walls of these vessels passively
3 expand and the receptors fire. Carotid sinus massage stimulat es the firing of baroreceptors, which then synapse in the medulla, lea ding to autonomic
changes. An increa se in firing of these receptors lea ds to a decrea se in heart rat e via slowed conduction through both the sinoatrial and atrioventricular
4 nodes as a result of an increa se in parasympathetic activity.
Synapse Parasympathetic nervous system Baroreceptor Carotid sinus Heart rate Atrioventricular node Medulla oblongata Autonomic nervous system
5
Carotid sinus massage Sinoatrial node Common carotid artery Receptor (biochemistry) Blood pressure
6
A is no t co rrect. 6 % cho se this.
7
The parasympathetic nervous system would increa se the PR interva l as a result of a decrea sed conduction velocity through the atrioventricular node.
8 Atrioventricular node Parasympathetic nervous system Nervous system Nerve conduction velocity

9 B is no t co rrect. 4 % cho se this.

10 The parasympathetic nervous system would decrea se conduction velocity through both the sinoatrial and atrioventricular nodes via vagal conduction .
Parasympathetic nervous system Atrioventricular node Vagus nerve Nervous system Sinoatrial node Nerve conduction velocity
11
c is no t co rrect. 8% cho se this.
12 Peripheral vascular resistance is altered via the action of the sympathetic nervous system . An increa se in peripheral va scular resistance would occur in a
13 stat e in which there is increa sed sympathetic activity; carotid massage prompts an increa se in parasympathetic activity. Parasympathetic activity will
reduce conduction velocity through the sinoatrial and atrioventricular nodes via va gal conduction ; additionally, parasympathetic activity will have a
14 vasodilatory effect on blood vessels.
Sympathetic nervous system Vascular resistance Autonomic nervous system Vasodilation Parasympathetic nervous system Atrioventricular node Nervous system
15
Vagus nerve Sinoatrial node Common carotid artery Blood vessel Carotid artery
16
D is no t co rrect. 3 % cho se this.
17 Ventricular contractility slightly decrea ses not increa ses. Ventricular contractility does not significantly respond to the parasympathetic nervous system
18 since innervation of the ventricular myocardium by cholinergic fibers is minimal. As a result, only a small decrea se of contractility occurs in the ventricles.
However; the parasympathetic nervous system has a significant negative inotropic (decrea se in contractility) effect in the atria .
• 19 Parasympathetic nervous system Cholinergic Cardiac muscle Inotrope Atrium (heart) Ventricle (heart) Contractility Nervous system Ventricular system Nerve

• 20
• 21 Botto m Li ne:

6
lock
s
Suspend
0
End Block
Item: 18of25 ~. , . M k <:] t> al ~· ~
QIO: 1038 .l. ar Previous Next lab 'Vfl1ues Notes Calculator

1
Bottom Line:
2
Carotid sinus massage stimulates the carotid baroreceptors, which leads to slowing of the heart rate via activation of the parasympathetic nervous
3 system.
4 Parasympathetic nervous system Baroreceptor Carotid sinus Carotid sinus massage Heart rate Bradycardia Nervous system Carotid artery Common carotid artery
Massage
5
6
7
I ill ;fi 1!1 I•J for year:[ 2017 ..
FI RST AI D FA CTS
8
9
FA17 p 285.3
10
Baroreceptors and chemoreceptors Receptors:
11
AFFERENT EFFERENT
• Aortic arch transmits via vagus nerve to sol itary nucleus of
12
medulla (responds to ! and t in BP).
13 • Ctuotid sinus (dilated region at carotid bifurcation) transmits via
14 glossophar)•ngcal nerve to solitary nucleus of medulla (responds
/ " Sympathetic to ! and t in BP).
15 chain
16 Baroreceptors:
17 • Hypotension-! arterial pressure .... l stretch .... l afferent
Para.sympathetic baroreceptor firing .... t efferent sympathetic firing and
18 vagus nerve
l efferent parasympathetic stimulation .... vasoconstriction,
• 19
Carotid sinus- - "• t HR, t contractility, t BP. important in the response to severe
• 20 baroreceptor Sympathetic hemorrhage.
Carotid body nerves • Carotid massage- t pressure on carotid sinus .... t stretch
• 21
rhPmnrPrPntnr I I
6
lock
s
Suspend
0
End Block
Item: 18 of 25 ~ 1 • M k -<:J 1>- Jil ~· !:';-~
QIO: 10 38 ..L ar Pre v ious Next Lab~lues Notes Calcula t o r
& &
1
2
FA17 p273.1
3 Cardiac output variables
4
Stroke vo lume Stroke Volume affected b) Contractility, SVCAP.
5 \ fterload, and Preload. A failing heart has l SV (systolic and/or diastolic
6 t SV with: dysfunction)
7 t Contractility (eg, anxiety, exercise)
• t Preload (eg, early pregnancy)
8
• l Afterload
9
Contract ility Contractility (and SV) t with: Contractility (and SV) l with:
10
• Catecholamine stimulation \'ia ~ 1 receptor: ~ 1 -blockade (l cA~ lP)
11 Ca 2+ channels phosphorylated - t Ca 2+ IJF with systolic dysfunction
12 entry - t Ca 2•-induced Ca 2• release and cidosis
13 t Ca 2+ storage in sarcoplasmic reticulum Hypoxia/hypercapnia (l Po, I t Pco,)
14 Phospholamban phosphor)lation - active Non-dihydropyridine Ca 2• ~hanncl blockers
Ca 2+ ATPase - t Ca 2+ storage in
15
sarcoplasmic reticulum
16 • t intracellular Ca2+
17 • l extracellular 1 a+ U activity of 1 a-t /Ca 2+
18 exchanger)
. 19 Digitalis (blocks Na+JK+ pump
.... t intracellular 1a+ .... l 1 a+/Ca 2•
• 20
exchanger acti\'ity .... t intracellular Ca 2•)
. 21
• .... . . . . . . .. ~; . . . _".,,.. ..... _ \\ /,..11 t.-,...,,. :,......., f,..,JI,.., .. .... I n.-..l n,..,..',. )..., •• •.

a
Lock
s
Suspend
8
End Bl ock
Item: 19 of 25 ~ 1 • M k -<:J 1>- Jil ~· !:';-~
QIO: 10 3 0 ..L ar Pre v ious Next Lab~lues Notes Calcula t o r
& &
1
The graph Illustrates a typical myocardial action potential. Each phase of the trace can be attributed to the conductance of ions through their
2 respective channels.

3 Ventricular muscle fiber


4
2
5
6
30rns
1----l
7
0
8
9
10
4
11

12
13 During phase 2 of the myocardial action potential, which of the following drugs has the most significant effect on the flow of ions through voltage-gated
channels?
14
15 A. Adenosine
16
B. Atropine
17
18
c. Nifedipine

• 19 o. Procainamide

• 20 E• Propranolol
• 21
• c: , ,......... r-o.-.-11

a
Lock
s
Suspend
8
End Bl ock
Item: 19 of 25 ~ 1 • M k -<:J 1>- Jil ~· !:';-~
QIO: 10 3 0 ..L ar Pre v ious Next Lab~lues Notes Calcula t o r

1 &
Ventricular muscle fiber &

2
2
3
4 30ms
~
5
0
6
7
8 4
9
10
During phase 2 of the myocardial action potential, which of the following drugs has the most significant effect on the flow of ions through voltage-gated
11 channels?
12
13 A. Adenosine
14 8. Atropine
15
c. Nifedipine
16
17
o. Procainamide

18 E. Propranolol
. 19
F• Verapamil
• 20
. 21

a
Lock
s
Suspend
8
End Bl ock
Item: 19 of 25 ~ 1 • M k -<:J 1>- Jil ~· !:';-~
QIO: 1030 ..L ar Previous Next Lab~lues Notes Calculat or
& &
1 Th e co rrect a n sw er i s F. 560/o chose this.
2 Verapamll Is a calcium channel blocker. During phase 2 of the tracing, calcium and potassium channels are open, and calcium influx into the cell matches
potassium efflux from the cell. The creates the plateau appearance of phase 2. Calcium channel blockers have the most impact on phase 2 compared with
3 other phases. Non-DHP calcium channel blockers (eg, verapamil) also depress conduction through the atrioventricular (AV) node.
4 Calcium channel blocker Verapamil calcium channel calcium Potassium Atrioventricular node Potassium channel

5 A i s not correct. 9 0/o chose t his.


Adenosine's prinapal effect is by inhibiting sinoatrial (SA) nodal, atrioventricular (AV) nodal, and His-Purkinje cell activity. Adenosine works to
6 hyperpolarize the cell via potassium channels and inhibiting L-type calcium channels, which decreases conduction velodty (a negative dromotropic
7 effect). Though 1t does have minor indirect effects on the myocardium, adenosine will not affect cal dum flow as much as verapamil.
Verapami Ca doac muscle Calcium Adenosine Hyperpolarization 'boology Dromotroooc L-type calcium channel Sinoatrial node Potassium channel
8 Voltage-dep.,ndent calcium channel Calcium channel Potassium Cavl.l
9 B i s n o t co rrect. 50/o chose this.
10 Atropine is a muscarinic antagonist that increases heart rate but does not affect the inotropic balance of the card iac cycle. It binds to the muscarinic
receptors in the sinoatrial (SA) and atrioventricu lar (AV) nodes, blocking aceychollne-mediated vagal activity. Atropine can be used to treat symptomatic
11 bradycardia.
12 Atropine Bradycardia Muscarinic antagonist Inotrope Receptor antagonist Heart rate Cardiac cycle Vagus nerve Sinoatrial node Muscarinic acetylcholine receptor
Antagonist
13
C is not co rrect. 14 0/o chose this.
14
Nifedipine Is a DHP calcium channel blocker. DH P calcium channel blockers are potent vasodilators but do not have as significant an effect on myocardial
15 action potential as non-DHP calcium channel blockers, such as verapamll .
Calcium channel blocker Nifedipine Action potential Verapamil Calcium Calcium channel Vasodilation
16
D is not co rrect . 100/o ch ose this.
17
Procalnamlde Is a sodium and potassium channel blocker and decreases how fast the action potential arises in phase o. It is a class l A drug along with
18 quinidine and disopyramide. In contrast, the class l B and l C drugs only block sodium channels.
Procainamide Action potential Disopyramide Quinidine Sodium Sodium channel Potassium
19
E i s n ot correct. 6 0/o ch ose this.
• 20
Propranolol is a 13-blocker, which increases the refractory period (phase 4) through slowing the conduction through the sinoatria l (SA) and atrioventricular
• 21 (AV) nodes.

a
Lock
s
Suspend
8
End Block
Item: 19 of 25 ~. I • M k <:] t> al ~· ~
QIO: 1030 .l. ar Previous Next lab 'Vfl1ues Notes Calculator

1 •
Procainamide is a sodium and potassium channel blocker and decrea ses how fast the action potential arises in phase 0. It is a class l A drug along with
2 quinidine and disopyramide. In contrast, the class l B and l C drugs only block sodium channels.
Procainamide Action potential Disopyramide Quinidine Sodium Sodium channel Potassium
3
E is not correct. 6% chose this.
4 Propranolol is a j3 -blocker; which increa ses the refractory period (phase 4 ) through slowing the conduction through the sinoatrial (SA ) and atrioventricular
5 (AV ) nodes.
Propranolol Refractory period (physiology) Refractory period (sex) Refractory Sinoatrial node
6
7
Bottom Line:
8
During the myocardial action potential, remember which channels are open and which are closed during ea ch phase. During phase 2, calcium influx
9 balances potassium efflux, which lea ds to the plat eau shape of the action potential. Calcium-channel blockers are effective during this phase of the
action potential.
10 Action potential Calcium Calcium channel Calcium channel blocker Potassium
11

12
13 I ill ;fi 1!1 I•J for year:[ 2017 ..
FI RST AID FA CTS
14
15
FA17p280.1
16
Myocardial action Also occurs in bundle of His and Purkinje fi bers.
17
potential
Phase 0 = rapid upstroke and depolarization-voltage-gated Ia+ channels open.
18
19 Phase 1 = initial repolarization -inactivation of voltage-gated a+ channels. Voltage-gated K+
channels begin to open.
0 20
Phase 2 =plateau-Cal+ inAux through voltage-gated Cal+ chan nels balances K+ efAux. Cal+
0 21
• . n , . r ..
6
lock
s
Suspend
0
End Block
Item: 19 of 25 ~ 1 • M k -<:J 1>- Jil ~· !:';-~
QIO: 10 3 0 ..L ar Pre v ious Next Lab~lues Notes Calcula t o r
& &
1
"Leak" currents
2
3
FA17 p 304.2
4
5
Calcium channel Amlodipine, cle\'idipine, nicardipine, nifedipine, nimodipinc (dihydropyridines, act on \<Jscular
blockers smooth muscle); diltiazem, ' erapamil (non-d ihydropyridines, act on heart).
6
MECHANISM Block voltage-dependent L-type calcium channels of cardiac and smooth muscle - l muscle
7
contractility.
8
Vascular smooth muscle-amlodipine =nifedipine > diltiazem > verapamil.
9 lleart-,·erapamil > diltiat.em > amlodipine = nifedipine (' erapamil =' entricle).
10 CliNICAl USE Dihydropyridines (except nimodipine): hypertension, angina (including Prinzmctal), Raynaud
11 phenomenon.
12 t\imodipine: subarachnoid hemorrhage (pre,·ents cerebral ,-asospasm).
t\icardipine, clevidipine: hypertensive mgency or emergency.
13
1'\on-dihydropyridines: h)'pertension, angina, atrial fibrillation/Autter.
14
ADVERSEEFFECTS Non-dihydropyridine: cardiac depression, AV block, hypcrprolactinemia, constipation.
15
Dihydropyridine: peripheral edema, Rushing, dizziness, gingival hyperplasia.
16
17
FA17p281 .1
18
Pacemaker action Occurs in the SA and AV nodes. Key differences from the ,·entricular action potential include:
19
potential
• 20 Phase 0 = upstroke-opening of' oltagc-galed Ca2+ channels. Past ,·oltage-gated Ia+ channels are
. 21 permanently inacti,-aled because of the less negative resting potent ial of these cells. Results in a slow

a
Lock Suspend
s 8
End Bl ock
Item: 20 of 25 ~ 1 • M k -<:J 1>- Jil ~· !:';-~
QIO: 4778 ..L ar Pre v ious Next Lab~lues Notes Calcula t o r
& &
1
A 67-year-old man presents to the emergency department because of acute chest pain that began more than 6 hours ago. ECG shows elevated ST-
2 segments in three consecutive leads. If a biopsy of the affected tissue were taken for experimental purposes after reperfusion following
catheterization, images similiar to the ones shown below would be seen.
3
4
5
6
7
8
9
10
11
I m age copyright ©2012 Turillazzi et at.; licensee BioMed Cent r al Ltd
12
13 What Is the best explanation for the change in normal histology?
14
15 A. Collagen deposits and fibrosis

16 B. Elevated intracellular calcium level


17
C. Inability to form cross-bridges between m yosin and actin
18
D. Macrophage-mediated resorption
19
• 20
. 21

a
Lock
s
Suspend
8
End Bl ock
Item: 20 of 25 ~ 1 • M k -<:J 1>- Jil ~· !:';-~
QIO: 4778 ..L ar Pre v ious Next Lab~lues Notes Calcula t o r
& &
1
2
Th e correct an sw er i s B. 5 4 0/o chose this.
3

·---
Contraction bands are among the first signs seen after
4 myocardial Infarction (MI) reperfusion and are thought to be

5
6
due to hypercontraction of the reperfused myocardium
adjacent to areas of myocyte necrosis resulting in
sarcolemmal rupture. Within the first 12 hours, these bands
.....-~ A ' 111 afuulua
11 ..,... IJ u 'u . . .
X

~l~
L!,...,
1

'r ~
........

...
11'11 - . .n 1 'ft)

are seen at the marg1n of the necrotic area. The bands are
7 caused by hypercontraction of the sarcomeres due to massive
calcium Influx. This is secondary to what is called "oxygen Release of calcium from . . .
8 paradox" or "calcium paradox," as illustrated in the image. intemllf ator"
This occurs after reperfusion and can lead to both release of
9
reactive oxygen species (ROS) and activation of
10 phosphollpases (PLs). ROS leads to peroxidation of lipid
membranes and increase in calcium permeability, leading to
11 elevated Intracellular calcium. In addition, PL activation leads •;;eo 1 t s•en
to release of calcium from intracellular stores. Both pathways
12 Induced by reperfusion result in build -up of intracellular
13 calcium levels, leading to hypercontraction (see diagram).
Myocyte Reactive oxygen species Myocardial infarction Cardiac muscle Lipid Calcium Sarcomere Necrosis Reperfusion injury Oxygen Infarction Phospholipase
14 Intracellular Semipermeable membrane Redox Lipid peroxidation
15 A is no t co rrect . 150/o cho se this.
16 Formation of fibrotic tissue following a myocardial infarction (MI) takes several weeks. This patient's symptoms have been present only for a few hours,
which Is not long enough for fibrosis to occur. If this patient presented several weeks to months following a myocardial infarction with new-onset
17 pericardia I chest pain, then post-cardiac injury syndrome, also known as Dressler syndrome, would be high in the differential. weeks to months Is typically
18 enough time for fibrosis to occur from the MI , but post-cardiac injury syndrome would not cause contraction bands.
Myocardial infarction Chest pain Fibrosis Dressier syndrome Pericardium Infarction
19
C i s n o t co rrect. 2 0 0/o chose this.
20 Massive calcium influx into the cytoplasm as a result of the ischemic event would have the opposite effect. Calcium removes the troponin-tropomyosln
. 21 Inhibition, thus all owing the cross-bridge between myosin and actin to form and contraction to occur.
• ("vtnnl.:ac:m A tin Mvnc.in r;:alrium Tc.rhomi;:.

a
Lock
s
Suspend
8
End Bl ock
Item: 20 of 25 ~. I • M k <:] t> al ~· ~
QIO: 4778 .l. ar Previous Next lab 'Vfl1ues Notes Calculator

1 pericardiaI chest pain, then post -cardiac injury syndrome, also known as Dressier syndrome, would be high in the differential. Weeks to months is typically
enough time for fibrosis to occur from the MI, but post -cardiac injury syndrome would not cause contra ction bands.
2 Myocardial infarction Chest pain Fibrosis Dressier syndrome Pericardium Infarction

3 c is not correct. 20 % chose this.


4 Massive calcium influx into the cytoplasm as a result of the ischemic event would have the opposite effect. Ca lcium removes the troponin -tropomyosin
inhibition, thus allowing the cross-bridge between myosin and actin to form and contra ction to occur.
5 Cytoplasm Actin Myosin Calcium Ischemia
6 Dis not correct. 11% chose this.
7 Macrophages are visible in large quantities 4 - 7 days after myocardial infarction and are critical to the remova l of necrotic tissue, referred to as "yellow
softening." This patient's symptoms have not been present long enough to show the presence of macrophages.
8 Myocardial infarction Macrophage Necrosis
9
10
Bottom Line:
11 Contraction bands are caused by massive calcium influx and result in hypercontraction of the sarcomeres.
12 Sarcomere Calcium

13
14
15
lijj ;fi IJ l•l for year:l 2017 ..
FI RST AI D FA CTS

16
17 FA17p294.1

18 Evolution of Commonly occluded coronary arteries: LAD > RCA > circu mflex.
19 myocardial infarction Symptoms: diaphoresis, nausea, vomiting, severe retrosternal pain, pain in left arm and/or jaw,
20
shortness of breath, fat igue.
• 21 TIME GROSS LIGHT MICROSCOPE COMPLICATIONS

6
lock
s
Suspend
0
End Block
Item: 21 of 25 ~ 1 • M k -<:J 1>- Jil ~· !:';-~
QIO: 3990 ..L ar Pre v ious Next Lab~lues Notes Calcula t o r
& &
1
A patient with atrial fibrillation is started on a drug after failing first-line therapies. His physician is careful to monitor for adverse effects such as
2 pulmonary fibrosis and thyroid abnormalities. The diagram illustrates the cardiac action potential.

3 Ventricular muscle fiber


4
2
5
6
30ms
1----l
7
0
8
9
10
4
11
12
During which phase does this drug have its most significant clinical effect?
13
14
A. Phase o
15
16 B. Phase 1

17 c. Phase 2
18
o. Phase 3
19
E. Phase 4
20
. 21

a
Lock
s
Suspend
8
End Bl ock
Item: 21 of 25 ~ 1 • M k -<:J 1>- Jil ~· !:';-~
QIO: 3990 ..L ar Previous Next Lab~lues Notes Calculat or
& &
1 The co rrect a n sw er i s o. 63% chose this.
2 Amlodarone is a class III antiarrhythmic and exerts its actions by blocking potassium channels. Potassium channels are important in all phases of the
cardiac action potential, but amiodarone's key effects are prolonging the action potential and the effective refractory period. Massive potassium efflux
3 causes the rapid repolarization seen in phase 3. Blocking potassium channels prolongs phase 3. Amiodarone does have some class I effects, but this Is not
4 Its main mechanism of action. Adverse effects of amiodarone include Interstitial pulmonary fibrosis, thyroid dysfunction (both hypothyroidism and
hyperthyroidism), hepatitis, blue-gray skin pigmentation in sun-exposed areas, and corneal deposits. Class IB drugs (lidocaine, mexiletine) shortens phase
5 3 repolarlzatlon in ventricular muscle.
Cornea •m•odaroo e Action potential Udocaine Hyperthyroidism Hypothyroidism cardiac action potential Pulmonary fibrosis Mexiletine Antiarrhythmoc agent
6
Potass• m c oa oe Refractory period (physiology) Potassium Hepatitis Thyroid Fibrosis Muscle Ventricle (heart)
7
A i s not correct. 1 0 % chose this.
8 Phase 0 represents rapid depolarization caused by the opening of voltage-gated sodium channels and subsequent sod ium influ x. Class Ia drugs (quinidine,
9 procalnamlde, d1sopyramide) and classIc drugs (flecainide, propafenone) slow phase 0 depolarization in ventricular muscle.
Shared adverse effects of class Ia drugs include thrombocytopenia and torsades de pointes due to increased QT interval. Qu inidine can cause cinchonism
10
(headache along with tinnitus). Procainamide can cause a reversible SLE-IIke reaction, whereas disopyramide can lead to heart failure. ClassIc drugs are
11 contraindicated in structural and ischemic heart disease because they are proarrhythmic (main side effect).

12 Torsades de pointes Flecainide Procainamide Oisopyramide Quinidine Tinnitus QT interval Coronary artery disease Propafenone Sodium Thrombocytopenia
Sodium channel Depolarization Ischemia Contraindication Cardiovascular disease Headache Heart failure Ventricle (heart) Cinchonism
13
6 is not co rrect. 60/o chose this.
14
Phase 1 Is Initial repolarization caused by inactivation of voltage -gated sodium channels and opening of voltage -gat ed potassium channels. The Initial
15 efflux of potassium causes a slight repolariza tion and is affected by amlodarone. However, in normal cardiac tissue the potassium efflux is quickly
counteracted by calcium influx, so amiodarone does not have a significant effect during phase 1.
16
Amiodarone Repolarization Potassium Voltage-gated potassium channel Sodium Potassium channel Sodium channel Calcium Efflux (microbiology)
17
C is n o t co rrect. 180/o ch ose this.
18 Phase 2 Is also known as the plateau phase. It consists of the opening of voltage-gated calcium channels that produce calcium influx, opposing the
repolarlzatlon effects of potassium efflux. The calcium influx also triggers opening of L type calcium channels in the sarcoplasmic reticulum and
19 subsequent myocyte contraction . Since the length of phase 2 is dictated by a balance between calcium influx and potassium efflux, amiodarone does
20 slightly prolong phase 2. However, since the plateau phase is short, its effects are much more dramatic and clinically relevant in phase 3.
Amoodarone Myocyte Sarcoplasmic reticulum Voltage-dependent calcium channe Potas~ium calcium Repolarization calcium channel Efflux (microbiology)
21 VoltaQe-Qated ion channel

a
Lock
s
Suspend
8
End Block
Item: 21 of 25 ~. I • M k <:] t> al ~· ~
QIO: 3990 .l. ar Previous Next lab 'Vfl1 ues Notes Calculator

1 c is no t co rrect. 18 % cho se this.


2 Phase 2 is also known as the plat eau phase. It consists of the opening of voltage -gat ed calcium channels that produce calcium influx, opposing the
repolarization effects of potassium efflux. The calcium influx also triggers opening of L type calcium channels in the sarcoplasmic reticulum and
3 subsequent myocyte contra ction . Since the length of phase 2 is dictat ed by a balance between calcium influx and potassium efflux, amiodarone does
slightly prolong phase 2 . However; since the plat eau phase is short, its effects are much more dramatic and clinically relevant in phase 3 .
4 Amiodarone Myocyte Sarcoplasmic reticulum Voltage-dependent calcium channel Potassium Calcium Repolarization Calcium channel Efflux (microbiology)
5 Voltage-gated ion channel

6 E is no t co rrect. 3 % cho se this.

7 Phase 4 is the resting potential after the myocyte contra ction . It occurs after the voltage -gat ed potassium channels opened during phases 2 and 3 close.
During phase 4 a different type of potassium channel, known as a potassium leak channel, operat es. This type of channel allows a very slow efflux of
8 potassium that essentially translat es into higher membra ne permeability to potassium. Although phase 4 involves potassium channels, this subtype is not
strongly affected by amiodarone. Class II drugs decrea se the slope of phase 4 for pacemaker cells
9
Side effects of class II drugs (j3 -blockers) include impotence, exacerbation of COPD and asthma, cardiova scular side effects (bradycardia, AV block, heart
10 failure), and CNS side effects (sedation, sleep alteration) . 13-Biockers can also mask the signs of hypoglycemia, which can be important in patients with
diabet es.
11
Tandem pore domain potassium channel Amiodarone Myocyte Hypoglycemia Bradycardia Resting potential Asthma Beta blocker Potassium channel Potassium
12
Diabetes mellitus Atrioventricular block Artificial cardiac pacemaker Central nervous system Chronic obstructive pulmonary disease Sedation Cardiac pacemaker
13 Heart failure Erectile dysfunction Cell membrane Voltage-gated potassium channel Circulatory system Voltage-gated ion channel
14
15
Bo tto m Line:
16
Amiodarone is a class III antiarrhythmic, which acts by prolonging stage 3 repolarization . Adverse effects include pulmonary fibrosis, thyroid dysfunction,
17 blue-gray discoloration of skin, and corneal deposits.
Cornea Amiodarone Pulmonary fibrosis Antiarrhythmic agent Thyroid Fibrosis
18
19
20
I ill ;fi 1!1 I•J f o r year:[ 20 17 ..
21 FI RST AID FA CTS

6
lock
s
Suspend
0
End Block
Item: 21 of 25 ~ 1 • M k -<:J 1>- Jil ~· !:';-~
QIO: 3990 ..L ar Pre v ious Next Lab~lues Notes Calcula t o r
I g y I I I p
1
Cornea Amiodarone Pulmonary fibrosis Antiarrhythmic agent Thyroid Fibrosis
2
3
4 lal;fii!!!•J tor year: 2017
FIRST AID FACTS
...
5
6
FA17p280.1
7
Myocardial action Also occurs in bundle of His and Purkinje fibers.
8
potential
9 Phase 0 =rapid upstroke and dcpolarization-\'oltage-gated 'a+ channels open.
10 Phase 1 = initial repolarization-inactivation of ,·oltage-gated I\ a+ channels. Voltage-gated K+
11
channels begin to open.
12 Phase 2 = plateau-Ca 2+ inAux through ,·ollage-gated CaZ+ channels balances K+ efAux. CaZ+
13
influx triggers Ca 2+ release from sarcoplasmic reticulum and myocyte contraction.
14 Phase 3 =rapid repolarization- massive K+ efnu x due to open ing of voltage-gated slow K+
15
channels and closure of voltage-g<~ted C<121 channels.
16 Phase 4 = resting potential- high K+ permeability through K+ channels.
17
In contrast to skeletal muscle:
18
Cardiac muscle action potential has a plateau, wh ich is due to Ca 2+ inAux and K+ efAux.
19 Cardiac muscle contraction requires Ca 2+ in nux from ECF to induce CaZ+ release from
20 sarcoplasmic reticulum (Ca 2+ -induced Ca 2+ release).
21 Cardiac myocytes are electrically coupled to each other by gap junctions.

a
Lock
s
Suspend
8
End Bl ock
Item: 21 of 25 ~ 1 • M k -<:J 1>- Jil ~· !:';-~
QIO: 3990 ..L ar Pre v ious Next Lab~lues Notes Calcula t o r
& &
1 FA17 p 309.2
2
Antiarrhythmics- Amiodarone, l butilide, Dofetilide, Sotalol. AIDS.
3 potassium channel
4 blockers (class Ill)
5 MECHANISM t AP duration, t ERP, t QT interval.
6 CliNICAL USE Atrial fibrillation, atrial Outler; ventricular
7 tachycardia (amiodarone, sotalol).
8 ADVERSE EFFECTS Sotalol-torsades de pointes, e\cessive p Remember to check PFTs, LFTs, and TFTs ''hen
9 blockade. using amiodarone.
lbutilide-torsades de pointes. Amiodarone is lipophilic and has class I, II, Ill ,
10
Amiodarone-pulmonarr fibrosis, and IV effects.
11
hepatotoxicity, hypothyroidism/
12 hyperthyroidism (amiodarone is 40% iodine by
13 weight), acts <lS hapten (corneal deposits, blue/
14 gray skin deposits resulting in photodermatitis),
neurologic effects, constipation, cardiovascular
15
effects (bradycardia, heart block, H ~').
16
Class Ill
17
OmV
18
Markedly prolonged
19 repolanzabOn (IK)

20
21

a
Lock
s
Suspend
8
End Bl ock
Item: 22 of 25 ~ 1 • M k -<:J 1>- Jil ~· !:';-~
QIO: 10 32 ..L ar Pre v ious Next Lab~lues Notes Calcula t o r
A A

2 A 55-year-old man came to cardiologist for a review of his condition. He has suffered an acute myocardial infarction 2 years ago, which is being
appropriately managed with medications. Currently, the patient denies chest pain, breathlessness, and leg swelling. o n exa mination, his temperature
3
was 37.2°C (98.96°F), heart rate 74 beats/minute, respiratory rate 14 breaths/minute, blood pressure 130/85 mm Hg. On auscultation of the chest,
4 lungs fields were clear, and cardiac sounds 1 and 2 were present with no murmurs or added sounds. No significant findings are found on the rest of physical
examination.
5
6
7
8
9
10

11

12
13
14
15
16
17
18
19 Using the Image as a guide, the order of conduction velocity in the normal heart, from fastest to slowest, is

20
A. 1; 2; 3
21
• 22 8. 1; 3; 2

a
Lock
s
Suspend
8
End Bl ock
Item: 22 of 25 ~ 1 • M k -<:J 1>- Jil ~· !:';-~
QIO: 10 32 ..L ar Pre v ious Next Lab~lues Notes Calcula t o r
A A

2
3
4
5
6
7
8
9
10
11
12 Usi ng the Image as a guide, the order of conduction velocity in the normal heart, from fastest to slowest, is

13
14 A. 1; 2; 3

15 8. 1; 3; 2
16
c. 2; 1; 3
17
o. 2; 3; 1
18
19 E. 3; 1; 2
20 F. 3; 2; 1
21
. 22 •
a
Lock
s
Suspend
8
End Bl ock
Item: 22 of 25 ~. I • M k <:] t> al ~· ~
QIO: 1032 .l. ar Previous Next Lab 'Vfll ues Notes Calculator

2 The correct answer is c. 44% chose this.


3 In the vignette image, 1 refers to the bundle of His, 2 is the Purkinje system, and 3 is the
atrioventricular (AV) node. Conduction velocity is a function of the size of the cells, the Structure of Conduction
Description
4 number of cells that make up the node, the number of gap junctions, and the composition Heart Velocity
of ion channels that dictat e a specific cell's action potential. The AV node is the slowest
5 point (0 .05 m/sec; as shown in the t able) in the chain of conduction in the heart, because Small cells, few gap
the cells in the node are small, have few gap junctions, and have a slow-rising, low - junctions, slow-rising and
6 AVnode 0.05 m/sec low-amplitude action
amplitude action potential. In contrast, the His-Purkinje system has very large cells with
potentials
7 many gap junctions and larger action potentials, crea ting a faster conduction velocity. The
bundle of His conducts at 0 .8 - 1 m/sec, and the Purkinje system conducts at 4 m/sec. Large cells, many gap
8 Bundle of His Action potential Gap junction Atrioventricular node Bundle of His 0.8-1 m/sec junctions, large action
potentials
9 Electrical conduction system of the heart Ion channel Nerve conduction velocity
Large cells, many gap
10 Purkinje fibers 4 m/sec junctions, large action
potentials
11
12 A is not correct. 14% chose this.
In the image in the vignette, 1 refers to the bundle of His, 2 is the Purkinje system, and 3 is the atrioventricular (AV) node. The His-Purkinje system
13 consists of specialized fibers with a large number of gap junctions that give it a faster conduction velocity than the either the AV node or bundle of His.
14 Bundle of His Atrioventricular node Gap junction Electrical conduction system of the heart Nerve conduction velocity

15 B is not correct. 7% chose this.


In the image in the vignette, 1 refers to the bundle of His, 2 is the Purkinje system, and 3 is the atrioventricular (AV) node. The Purkinje system has a
16 faster conduction velocity than both the atrioventricular node and bundle of His. This is because the Purkinje system consists of specialized fibers whose
17 numerous gap junctions and voltage-gat ed sodium channels allow for quick conduction of an impulse.
Atrioventricular node Bundle of His Gap junction Sodium channel Nerve conduction velocitv Sodium Voltage-gated ion channel
18
D is not correct. 13% chose this.
19 In the vignette image, 1 refers to the bundle of His, 2 is the Purkinje system, and 3 is the atrioventricular (AV) node. The bundle of His has a faster
20 conduction velocity than the atrioventricular node. Conduction velocity is slowed at the AV node, and action potentials there are reliant on slow L-type
calcium channels instea d of fast sodium channels, which are responsible for the quick upward slope of the action potential in other area s of the heart .
21 Atrioventricular node Bundle of His Action potential Sodium channel Sodium Voltage-dependent calcium channel Calcium channel Calcium l-type calcium channel

22 E is not correct. 15% chose this.

6
lock
s
Suspend
0
End Block
Item: 22 of 25 ~. I • M k <:] t> al ~· ~
QIO: 1032 .l. ar Previous Next Lab 'Vfll ues Notes Calculator
r '
2
D is not correct. 1 3% chose this.
3 In the vignette image, 1 refers to the bundle of His, 2 is the Purkinje system, and 3 is the atrioventricular ( AV) node. The bundle of His has a faster
conduction velocity than the atrioventricular node. Conduction velocity is slowed at the AV node, and action potentials there are reliant on slow L-type
4 calcium channels instea d of fast sodium channels, which are responsible for the quick upward slope of the action potential in other area s of the heart .
5 Atrioventricular node Bundle of His Action potential Sodium channel Sodium Voltage-dependent calcium channel Calcium channel Calcium l-type calcium channel

6 E is not correct. 15% chose this.


In the vignette image, 1 refers to the bundle of His, 2 is the Purkinje system, and 3 is the atrioventricular ( AV) node. The bundle of His and Purkinje
7 system has a faster conduction velocity than the atrioventricular node.
8 Atrioventricular node Bundle of His Nerve conduction velocity

9 F is not correct. 6 % chose this.


In the vignette imagem, 1 refers to the bundle of His, 2 is the Purkinje system, and 3 is the atrioventricular node. The bundle of His and the Purkinje
10 system have faster conduction velocities than the atrioventricular node.
11 Atrioventricular node Bundle of His Nerve conduction velocity

12
13 Bottom Line:

14 The speed of conduction from fastest to slowest ranks from Purkinje (4 m/s) > atria > ventricles> bundle of His (0 .8 - 1 m/s), and tied velocities of AV
and SA nodes (0 .05 m/s).
15 Bundle of His Atrium (heart) Ventricle (heart) Electrical resistivity and conductivity

16
17
18 Iii I;fi 1!1 I•J for year:l 2017 .. ]
FI RST AID FA CTS .
19
20
21
FA17p280.1
22 .. . •• "
, ........ 1 • • ,- ,

6
lock
s
Suspend
0
End Block
Item: 22 of 25 ~ 1 • M k -<:J 1>- Jil ~· !:';-~
QIO: 10 32 ..L ar Pre v ious Next Lab~lues Notes Calcula t o r
A A

2
FA17 p281 .1
3
Pacemaker action Occurs in the SA and AV nodes. Key differences from the ,·entricular action potential include:
4
potential
5 Phase 0 =upstroke- opening of \Oltage-gated Ca 2+ channels. Fast mltage-gated . a+ channels are
6
permanent!) inacti,<lted because of the less negative resting potential of these cells. Results in a slow
conduction ,·eJocity that is used by the A node to prolong transmission from the atria to ventricles.
7
Phases I and 2 are absent.
8
9 Phase 3 = inactivation of the Ca2• channels and t acti,ation of K+ channels .... t K• efAux.
10 Phase 4 =slow spontaneous diastolic depoh1rization due to Ir ("funny current"). Ir channels
11 responsible for a slow, mixed i a•tK• inward current; different from 1,3 in phase 0 of ventricular
action potential. Accounts for automaticity of SA and A nodes. The slope of phase 4 in the SA
12
node determines I IR. C h/adenosine l the rate of diastolic depolarization and l IIR, while
13
catecholamines t depolarization and t II R. S) mpathetic stimulation t the chance that Ir channels
14 are open and thus t HR.
15
16 0 ------------------------- -- ------------·
17
~
-20 '"
PhastO '•
Phase 3
18 -~
= -40
'i:
19
-60
20
-80
21 100 rMeC

22 •
a
Lock Suspend
s 8
End Bl ock
Item: 23 of 25 ~ 1 • M k -<:J 1>- Jil ~· !:';-~
QIO: 10 20 ..L ar Pre v ious Next Lab~lues Notes Calcula t o r
A A

3 The following graph is a depiction of the Starling curve showing the relationship between cardiac output and ventricular end diastolic volume In a
patient with congestive heart failure .
4
5
6 '5
~ 3.0
7
..'E
0
0
8
9
..
u
2.0

10
11 0 2 4 6 8 10 12
End-diastolic volume
12
13 If this patient Is treated with a positive inotrop ic agent, the Starling curve would do which of the following?
14
15 A. Flatten out
16 8. Shift down and left
17
c. Shift down and right
18
19 o. Shift right only

20 E. Shift up and left


21
F. Shift up and right
22
. 23 •
a
Lock
s
Suspend
8
End Bl ock
Item: 23 of 25 ~ 1 • M k -<:J 1>- Jil ~· !:';-~
QIO: 10 20 ..L ar Prev ious Next Lab~lues Notes Calculat o r

A A

3 Th e co rrect a n sw er i s E. 720/o chose this.


4 A positive Inotropic agent would increase the contractility of the heart, causing both stroke volume and cardiac output to increase at any given end-
diastolic volume. Therefore the curve would shift up and to the left.
5 Inotrope Stroke volume End-diastolic volume cardiac output Contractility Stroke

6 A i s not correct. 2 0/o chose t his.

7 The Starling curve does not change its shape in response to inotropic agents; rather, it sh1fts to the left and upward.
Inotrope
8
B i s not correct. 4 % chose this.
9
A shift down and to the left indicates a very low end -diastolic volume and cardiac output, which would occur in instances of decreased blood volume such
10 as In severe hemorrhage.
Cardiac output End-diastolic volume Bleeding Hypovolemia Blood volume
11
C i s not co rrect . 4 % chose this.
12 A shift of the Starling curve down and to the right would indicate an Increasingly falling heart. A positive inotropic agent would affect the heart In the
13 opposite manner and ameliorate the effects of heart failure.
Inotrope Heart failure
14
0 is not co rrect. JO/o chose t his.
15
The Starling curve shifts left, not right, as end-diastolic volume decreases. A decrease in end-diastolic volume will decrea se preload; cardiac myocytes will
16 stretch less and therefore will not contract as forcefully, leading to a decrease In stroke volume .
End-diastolic volume Stroke volume Cardiac muscle Preload (cardiology) Cardiac muscle cell
17
F is not co rrect . 150/o chose this.
18 A shift up and to the right would indicat e that there is an extremely high cardiac output along with a high end-diastolic volume . This would occur In the
19 presence of an extremely high blood volume, not an increase in contractility.
End-diastolic volume Cardiac output Contractility
20
21
Bottom Lin e:
22
The relationship between end-diastolic volume and increased force generation is known as the Frank-Starling relationship. understand how various
23 • druas and Datholoaies can affect the curve. For instance. in this case. a positive lnotrope shifts the curve UD and to the left because increased

a
Lock
s
Suspend
8
End Block
Item: 23 of 25 ~. I • M k <:] t> al ~· ~
QIO: 1020 .l. ar Previous Next lab 'Vfl1ues Notes Calculator
• presence of an extremely high blood volume, not an increase in contractility.
3 End -diastolic volume Cardiac output Contractility
4
5
Bottom Line:
6 The relationship between end-diastolic volume and increased force generation is known as the Frank-Starling relationship. Understand how various
7 drugs and pathologies can affect the curve . For instance, in this case, a positive inotrope shifts the curve up and to the left because increased
contractility increases the cardiac output and decreases the end-diastolic volume .
8 Inotrope End -diastolic volume Cardiac output Contractility

9
10
11 I iii I;fi 1!1I•J for year:l 2017 ..
FIRST AID FAC T S
12
13 FA17p274.1
14
Starling curve Force of contraction is proportiona 1 to end-
15 diastolic length of cardiac muscle fiber
16 (preload).
17 t contractility with catecholamines, positive
inotropes (eg, digoxin}.
18
l contractilit}' with loss of myocardium (cg, Ml),
19 HF +digoxin ~-blockers (acutely}, non-dihydropyridine Ca 2+
20
.-l,,.---
- .......
...
channel blockers, dilated cardiomyopathy.
21 HF
22
23 •

6
lock
s
Suspend
0
End Block
Item: 24 of 25 ~ 1 • M k -<:J 1>- Jil ~· !:';-~
QIO: 1004 ..L ar Pre v ious Next Lab~lues Not es Calcula t o r
A A

4 A 65-year-old man presents to the emergency department with chest pain that started abruptly after he climbed a flight of stairs. The emergency
physician orders an exercise stress test .
5
6
What Is the primary mode of compensation in response to increased myocardial demand during exercise stress testing?
7
8 A. Increased coronary artery diameter and increased coronary blood flow
9
B. I ncreased coronary artery diameter and increased oxygen extraction
10
11 c. Increased coronary artery resistance and decreased metabolite production

12 D. Increased coronary artery resistance and decreased oxygen extraction


13
E. Increased coronary artery resistance and increased oxygen extraction.
14
15
16
17
18
19
20
21
22
23
. 24 •
a
Lock
s
Suspend
8
End Bl ock
Item: 24 of 25 ~. I • M k <:] t> al ~· ~
QIO: 1004 .l. ar Previous Next lab 'Vfl1ues Notes Calculator

4 The co rrect a nswer is A. 64% cho se this.


An increa se in myocardial contractility due to exercise lea ds to increa sed oxygen consumption by the cardiac muscle, causing local hypoxia and increa sed
5 local levels of adenosine and C0 2 . This causes vasodilation of the coronary arterioles, which in turn allows for a compensatory increa se in coronary blood
flow . Oxygen extraction from heart muscle is maximized and thus is not the limiting factor in oxygen supply. Increa sed oxygen demand can be met only
6
by increa sing blood flow.
7 Hypoxia (medical) Vasodilation Cardiac muscle Hypoxia (environmental) Oxygen Contractility Adenosine Arteriole Blood flow Muscle Ejection fraction

8 B is no t co rrect. 21% cho se this.


9 Vasodilation occurs during exercise, increa sing coronary artery diamet er and thus blood flow . However; oxygen extraction is not increa sed. Compared with
most organs of the body, oxygen extraction by the heart is relatively high. It extracts optimal levels of oxygen at rest and therefore relies on the
10 increa sed coronary blood flow provided by vasodilation for further increa ses in oxygen supply. It does not have the ability to further increa se its oxygen
supply through extraction .
11 Vasodilation Oxygen Coronary circulation Blood flow
12 c is no t co rrect. 2% cho se this.
13 During exercise, coronary artery resistance decrea ses by hypoxia -induced va sodilation . Blood flow through the coronary circulation is controlled almost
entirely by local met abolites, with sympathetic innervation playing only a minor role. The most important local met abolic factors are hypoxia (decrea sed
14 oxygen) and increa sed adenosine, NO, and C0 2 . During periods of exercise and increa sed contractility of the heart, the production of adenosine increa ses,
but oxygen levels decrea se.
15
Hypoxia (medical) Coronary circulation Vasodilation Hypoxia (environmental\ Oxygen Adenosine Contractility Sympathetic nervous system Blood flow Metabolite
16 Nerve
17 D is no t co rrect. 4 % cho se this.
18 The heart cannot increa se or decrea se its oxygen extraction . Even if this were the primary means of supplying the extra demand for oxygen, the heart
would respond by increa sing oxygen extraction, not decrea sing it. In addition, exercise lea ds to vasodilation and, therefore, an increa se in coronary artery
19 diamet er; not a decrea se.
Vasodilation Oxygen Coronary circulation
20
21 E is no t co rrect. 9 % cho se this.
A decrea se in the diamet er of the coronary arteries (ie, increa sed coronary artery resistance) will decrea se, rather than increa se, coronary blood flow and
22 thus delivery of oxygen to the myocardium . Optimal cardiac oxygen extraction occurs at rest. The myocardium is unable to extract more oxygen during
exercise.
23
Cardiac muscle Coronary circulation Oxygen Artery
24

6
lock
s
Suspend
0
End Block
Item: 24 of 25 ~. I • M k <:] t> al ~· ~
QIO: 1004 .l. ar Previous Next lab 'Vfl1ues Notes Calculator

4 D is not correct. 4 % chose this.


5 The heart cannot increa se or decrea se its oxygen extraction . Even if this were the primary m ea ns of supplying the extra dem and for oxygen, the heart
would respond by increa sing oxygen extraction, not decrea sing it. In addition, exercise lea ds to vasodilation and, therefore, an increa se in coronary artery
6 diam et er; not a decrea se.
Vasodilation Oxygen Coronary circulation
7
E is not correct. 9 % chose this.
8
A decrea se in the diam et er of the coronary arteries (ie, increa sed coronary artery resistance ) will decrea se, rather than increa se, coronary blood flow and
9 thus delivery of oxygen to the myocardium . Optimal cardiac oxygen extraction occurs at rest. The myocardium is unable to extract more oxygen during
exercise.
10
Cardiac muscle Coronary circulation Oxygen Artery
11

12
Bottom Line:
13
Myocardial oxygen balance is det ermined by the ratio of oxygen supply to oxygen dem and, and a reduced oxygen supply: dem and ratio is the cause of
14 angina in coronary artery disea se. The oxygen supply can be increa sed by increa sing either arterial oxygen content or coronary blood flow. Apart from
hypoxia, coronary va sodilation also occurs with increa sed levels of adenosine, NO, and C0 2.
15 Hypoxia (medical) Vasodilation Coronary artery disease Hypoxia (environmental) Oxygen Angina pectoris Adenosine Coronary circulation
16
17

18 lijl;fiiJI•l toryear:[2017 • ]
FI RST AI D FA CTS
19
20
FA17 p273.1
21 Cardiac output variables
22 Stroke volume Stroke Volume affected by Contractility, SVCAP.
23 Afterload, and Preload. A failing heart has ~ SV (systolic and/or diastolic
24 t SV with: dysfunction)

6
lock
s
Suspend
0
End Block
Item: 24 of 25 ~ 1 • M k -<:J 1>- Jil ~· !:';-~
QIO: 1004 ..L ar Pre v ious Next Lab~lues Not es Calcula t o r

4
LV compensates for t afterload by thickening Chronic h) pertension (t MAP) - L
5
(hypertrophy) in order to ! wall tension. h) per trophy.
6
Ejection fraction EF' = SV = EDV- ESV EF l in systolic HF.
7 ~ ED EDV EF normal in HF with presen ed ejection
8 Left \·entricular EF is an index of \'entricular fraction (HFpEF').
9 contractility.
10
11 FA17 p 286.2

12 Autoregulation How blood Row to an organ remains constant over a wide range of perfusion pressures.
13 ORGAN FACTORS DETERMINING AUTOREGULATION
14 Heart Local metabolites (vasodih1tory): adenosine, Note: the pulmonary vasculature is unique in
15 0 , C0 2, ! 0 2 that hn>oxia causes vasoconstriction so that
16 Brain Loca l metabolites (vasoclilatorr): C0 2 (pi I) only well-ventilated areas arc perfused. In
other organs, hypoxia causes vasodilation.
17 Kidneys Myogenic and tubuloglomerular feedback
18 Lungs Hypoxia causes vasoconstriction
19 Skelet al muscle Local metabolites during exercise: lactate, C0 2, JJ+, Adenosine, Lactate, K+ (C IIALK)
20 adenosine, K+, J-l+, C0 2
21 At rest: srmpathetic tone
22 Skin Srmpathetic stimulation most important
23
mechanism for temperature control
24 •
a
Lock
s
Suspend
8
End Bl ock
Item: 25 of 25 ~ 1 • M k -<:J 1>- Jil ~· !:';-~
QIO: 10 49 ..L ar Pre v ious Next Lab~lues Not es Calcula t o r
A A

5 A 35-year-old male is scheduled for a standard stress test after being evaluated for chest pain. An ECG at the time of presentation was found to be
normal. He has never smoked and has no family history of cardiovascular disease. His BMI is 24, and he leads a sedentary lifestyle. A cardiac
6
pressure-volume tracing for this patient appears to be normal, with phases I-IV correspondingly labeled.
7
8
9 110 - ..._
10 Ill
Cl
11 I
E
12 E
13 ...
II)

:I IV II
1/)
14 1/)
20 -
15 ...
II)

16
17
18
19 I I
40 80 120 160
20
Volume, cc
21
22
Using the diagram as reference, during which phase are left atrial (LA) and left ventricular (LV) pressures most similar in a person with normal cardiac
23 function?

24
A. Phase I
• 25

a
Lock
s
Suspend
8
End Bl ock
Item: 25 of 25 ~ 1 • M k -<:J 1>- Jil ~· !:';-~
QIO: 10 49 ..L ar Pre v ious Next Lab~lues Not es Calcula t o r

5 Cl Ill
:I:
6 E
7
E
8
....:::J
Gl
IV II
Ill
9 Ill 20 -
....
Gl
10 CL.

11
12
13
I I
14 40 80 120 160
15 Volume, cc
16
17 Using the diagram as reference, during which phase are left atrial (LA) and left ventricular (LV) pressures most similar in a person with normal cardiac
function?
18
19
A. Phase I
20
B. Phase 11
21
22 c. Phase m
23 o. Phase IV
24
• 25 •

a
Lock
s
Suspend
8
End Bl ock
Item: 25 of 25 ~. I • M k <:] t> al ~· ~
QIO: 1049 .l. ar Previous Next lab 'Vfl1ues Notes Calculator

5 The co rrect a nswer is A. 57% cho se this.


Ventricular filling (phase I in the image) is the period between mitral valve opening and closing . During the period that the mitral valve is open,
6
ventricular pressure remains roughly equal to atrial pressure. Technically, the pressures in the atrium and ventricle are not equal as soon as the mitral
7 valve opens (LV pressure < LA pressure); however; as blood flows across the va lve the pressure equalizes and results in LV pressure > LA pressure before
the valve closes. Thus on average, the pressure can be considered equal.
8 Mitral valve Ventricle (heart) Atrium (heart) Ventricular system
9 B is no t co rrect. 13% cho se this.
10 Isovolumetric contraction (phase II in the image) is the period between mitral valve closing and aortic valve opening. The cardiac musculature contracts
against the closed aortic valve to drastically elevat e ventricular pressure. This patient has a normal aortic valve.
11 Isovolumetric contraction Aortic valve Mitral valve Ventricular pressure
12 c is no t co rrect. 12% cho se this.
13 Ventricular ej ection (phase III ) is the period between aortic valve opening and closing . Volume falls precipitously as blood rushes into the aorta and LV
pressure increa ses toward the end of ej ection to push the remaining blood into the aorta.
14 Aorta Aortic valve Clinical trial Ventricle (heart) Ventricular system Valve
15 D is no t co rrect. 18% cho se this.
16 Isovolumetric relaxation (phase IV) is the period in which both the aortic and mitral va lves are closed, thus keeping ventricular volume constant. This
phase ends when the ventricular pressure falls below the level of the atrial pressure, and the mitral valve opens to allow filling .
17 Mitral valve Valve
18
19
Botto m Li ne:
20 The cardiac cycle has four phases: (I ) ventricular filling, (II ) isovolumetric contraction, (III ) ventricular ej ection, and (I V) isovolumetric relaxation . For the
21 t est, you need to know which valves are open and which are closed during ea ch phase of the cycle.
Cardiac cycle Isovolumic relaxation time Isovolumetric contraction Ventricle (heart)
22
23
24
lijj ;fi IJ l•l f o r yea r:l 20 1 7 ..
FIRST AID FAC T S
25

6
lock
s
Suspend
0
End Block
Item: 25 of 25 ~. I • M k <:] t> al ~· ~
QIO: 1049 .l. ar Previous Next lab 'Vfl1ues Notes Calculator

5
6
Bottom Line:
7
The cardiac cycle has four phases: (I) ventricular filling, (II) isovolumetric contraction, (III) ventricular ej ection, and (IV) isovolumetric relaxation. For the
8 test, you need to know which valves are open and which are closed during each phase of the cycle .
Cardiac cycle Isovolumic relaxation time Isovolumetric contraction Ventricle (heart)
9
10
11
lijj ;fi IJ l•l for year:l 2017 ..
FI RST AI D FAC TS
12
13
FA17 p276.1
14
Pressure-volume loops and cardiac cycle The black loop represents normal card iac
15
physiology.
16
t Afterload
17 I (Aortic pressure)
I SV Phases- left ventricle:
18
I ESV 0 lsovolumetric contraction - period
19 between mitral valve closing and aortic
valve opening; period of highest 0 2
20
consumption
21 opens E} Systolic ejection-period between aortic
22
Pr lo d valve opening and closing
I SV E) lsovolumetric relaxation -period bet ween
23 aortic valve closing and mitral valve
Mitral
24 valve open •ng
25
closes 0 Rapid fi ll ing-period just after mitral
urtluo "'''o";,.,.n

6
lock
s
Suspend
0
End Block
Item: 25 of 25
QIO: 10 49
~
..L
1 • Ma r k -<:J
Pre v ious
I>
Next Lab
fJ lues
£!1}>'

Not es
!!":-~
Calcula t o r
A
.. ~ . .
A

5 0 01 0.2 0.3 0.4 0.5 0.6 0.7 0.8 tamponade.


Time (se<)
6
7
8 FA17 p 275.1

9 Cardiac and vascular function curves


10 O i inotropy
11 / - ------ c
3
E) i volume. venous tone
.............. ..
...........
/
--- ---- -----
12
13 t---. ~
I C.Vdaac functJon curve
~

~:·-
',•
.. ---
••
.
'

14
/ NOiffllA ~ - - - - - - ••
••
15 / ~ E) .!.1notropy •
I / ~
••
••
••
16 I / <-~,.. •• ••
••
Median ~~" •
1 / 0 .J. volume\
••
17 / systemic '1>~ • ••
18
I.'/ pressure~~ venous tone '.
••
••
••..._

19 RA nrp~~11rP or FnV RA pressure or EDV RA pressure or EDV

20 Intersection of curves= operating point of heart (ie, venous return and CO arc equa l).
21
GRAPH EFFECT EXAMPlES
22
23
e lnotropy Changes in contractility - altered CO for a 0 Catccholamines, digoxin @
given RA pressure (preload). E) Uncompensated HF, narcotic O\'erdose,
24
sympathetic inhibition e
25 _ _: ____ ) _ .__: __ ~·- ' ----- - -
I"'L _____ : _ ---··· • --·
~ --

a
Lock
s
Suspend
8
End Bl ock
Item: 25 of 25 ~ 1 • M k -<:J 1>- Jil ~· !:';-~
QIO: 10 49 ..L ar Pre v ious Next Lab~lues Not es Calcula t o r
A
- .. - . - A

5 co.
6 Changes often occur in tandem, and may be reinforcing (eg, exercise f inotropy and ! TPR to maximize CO) or
7 compensatory (eg, HF ! inotropy ..... fluid retention to t preload lo maintain CO).
8
9 FA17 p 278.1
10
Auscultation of the heart
11
12
Where to bsten. APT M
13
Aortic area:
14 Systolic murmur Pulmonic area:
15 AortJC stenos1s Systolic ejection murmur
Flow murmur Pulmon•c stenos1s
16 (1!9. phys1otog1c murmur) Flow murmur
AortJC valve sclerosiS
17 Tricuspid area:
Left sternal border: --~--------5-----~~--
18 Holosystolic murmur
0iastolic murmur Tricuspid regurgitation
19 AOrtiC r!!9urgitat10n Ventncular septal defect
Pulmon1c r!!9urg1tatlon Diastolic murmur
20 Tricuspld stenos1s
SystoliC murmur
21 HypertrophiC Atnal septal defect (i llow

22
cardiomyopathy ®- across tr1Cusp1d valve)

- - -- - - - - - Mitral area (apex):


23 - Aonic
Holosystolic murmur
- Pulmornc
24 - Tncusp.d M1tral r!!9urg1tat1on
M•uat SystoUc murmur
25 • M1tral valve orotaose

a
Lock
s
Suspend
8
End Bl ock

You might also like